arrow-downcheckdocdocxfbflowerjpgmailnoarticlesnoresultpdfsearchsoundtwvkxlsxlsxyoutubezipTelegram
Пролегомены ко всякой будущей теории истины
Текст подготовили: Артём Беседин, Александр Беликов, Андрей Зданевич, Евгений Логинов, Андрей Мерцалов, Артём Юнусов
Художники:   Наира Кочинян, Наталия Викторова, Сеня Уткин

Исследовательская группа в составе Беседина А.П., Беликова А.А., Зданевича А.В., Логинова Е.В., Мерцалова А.В. и Юнусова А.Т. публикует вторую часть материалов следователя, занимавшегося так называемым «делом Берлиоза». Первая часть, опубликованная нами ранее, содержала материалы, касающиеся печально знаменитого случая на Патриарших прудах *. В тех документах объект рассуждал о так называемых «доказательствах бытия божия». В публикуемых же ниже текстах, на первый взгляд, нет прямой религиозной закваски. Они посвящены якобы сугубо теоретической проблеме: природе истины. Однако реакционный характер этих «рассуждений» будет понятен всякому, кто знаком с подлинно научным методом философии и освоил хотя бы азы диалектики. Публикуемые материалы состоят из пяти частей. Часть, названная издателями нулевой, представляет собой вводное размышление, в котором можно видеть то место, где подвернулась нога рассуждения объекта, где началось его падение в метафизику. Это т.н. проблема истинностных значений, понятая автором метафизически. В этой же части он повествует о своей работе в архиве д. Кураева. В первой части публикуемых материалов автор рассуждает о проблеме истины в общем виде, сохраняя еще внешние атрибуты академической якобы нейтральности. Во второй части — все маски сброшены. Тут мы находим реакцию не только по содержанию, но и по форме: диалог, да еще и с участием библейских персонажей. Авторство и происхождение этой части представляет собой загадку. В науке существует четыре гипотезы. Первая: авторство принадлежит самому следователю. Этому противоречат эмоциональные пометки на полях в некоторых частях рукописи, сделанные рукой следователя, в которых порой выражается изумление или несогласие с содержанием текста. Три другие гипотезы исходят из того, что рукопись была взята следователем из архива д. Кураева, но расходятся в вопросе о ее авторстве. Согласно второй версии, автором второй части был сам д. Кураев. Третья утверждает, что авторство рукописи восходит к писателю О.Э. Мастеру, тогда как д. Кураев лишь пытался восстановить этот текст на основании неизвестно как им приобретенных в свое распоряжение архивных материалов и показаний И.Н. Понырева, с которым Мастер общался в психиатрической лечебнице А.Н. Стравинского. Наконец, четвертая версия выглядит совсем фантастической: ее сторонники, основываясь на известных со слов Понырева утверждениях Мастера, полагают, что архетипом рукописи является обнаруженный Мастером в подвальных фондах библиотеки имени В.И. Ленина текст средневекового ученого Герберта Аврилакского, который Мастер только пытался расшифровать и перевести на русский. Третья часть несомненно написана рукой ныне бывшего следователя и представляет собой своеобразное заключение к предыдущей части. Наконец, в качестве приложения мы публикуем отдельный лист, найденный в архиве следователя, который представляет собой, похоже, часть конспекта, видимо, использовавшегося при работе над рукописью, способный пролить свет на самые темные ее места.

§0. Истинностные значения

Зачем он приехал в Москву? Из показаний свидетеля Понырева И.Н., профессора, сотрудника Института философии АН СССР, следствию известно, что подозреваемый Воланд, предположительно профессор, историк по специальности, интурист из капстран, в разговоре с ним, Поныревым, и покойным Берлиозом М.А., редактором журнала «Богоборец» и председателем правления МАССОЛИТа, сообщил о себе следующее: «специализируюсь на папской власти». [1] Он также якобы показал Поныреву и Берлиозу, по сообщению первого, приглашение приехать в Москву для консультации. Вызванный вновь свидетель Понырев не смог припомнить никаких новых подробностей.

В пасквиле Булгакова М.А., антисоветски настроенного писателя, Воланд излагает весьма конкретную цель прибытия в советскую столицу: «Тут в государственной библиотеке обнаружены подлинные рукописи чернокнижника Герберта Аврилакского, десятого века, так вот требуется, чтобы я их разобрал». Устроенная очная ставка между профессором Поныревым и писателем Булгаковым показала: сведения, излагаемые писателем, подтверждаются показаниями профессора. В ответ на вопрос о том, почему до очной ставки Понырев не сообщал эти сведения следствию, профессор сослался на забывчивость. Слабость его памяти удостоверена специалистами психологического диспансера имени А.Н. Стравинского. На вопрос о том, откуда ему известны подробности упомянутой беседы на Патриарших, писатель Булгаков сообщил, что получил их от Андрея Вячеславовича Кураева, бывшего преподавателя кафедры научного атеизма Московского государственного университета имени М.В. Ломоносова. Не имея, ввиду известных печальных обстоятельств, возможности допросить известного ученого, следствие обратилось к его архиву, хранящемуся на указанной кафедре в ведомстве профессора В.В. Винокурова. В архиве ученого обнаружилось большое количество литературы мистического характера (что легко объяснимо, учитывая научные интересы Кураева), а также дневниковые записи. Из последних ясно, что в рамках научных исканий доцент Кураев анализировал бред психбольных, желая проверить истинность гипотезы буржуазного «психолога» Юнга о структурном подобии бреда и т.н. «литературы откровений». Это исследование привело д. Кураева в клинику имени А.Н. Стравинского, где он имел общение с будущим проф. Поныревым, который пребывал там еще под именем поэта Ивана Бездомного. Так как произошло это вскоре после известных происшествий, д. Кураев вполне мог получить от Понырева достоверную информацию. И в самом деле, в архиве обнаруживаются следы интереса ученого к фигуре Герберта Аврилакского. Возможно, что д. Кураев дополнительно консультировался с Жирмунским В.М., профессором филологии Ленинградского государственного университета, видным специалистом в области сравнительного литературоведения. Из записей д. Кураева следует, что при этой встрече присутствовал малолетний тогда внук ленинградского ученого Андрей. Вызванный для дачи показаний Андрей Аствацатуров, писатель и специалист по буржуазной литературе ХХ века, подтвердил, что указанное сношение имело место. Из записей д. Кураева и показаний Аствацатурова следует, что профессор Жирмунский сообщил московскому ученому, что названный Герберт Аврилакский был так называемым «римским папой», также известным под псевдонимом «Сильвестр II», жившим в Х веке нашей эры. Жизнь указанного Герберта стала предметом средневековых легенд, которые легли в основание предания о докторе Фаусте, который продал душу так называемому «дьяволу».

Эту информацию, судя по записям, ученый сверил с трудами московской ученой-историка-медиевиста Виолетты Павловны Гайденко и московского же специалиста по свету Александра Михайловича Шишкова. На основании полученной информации д. Кураев сделал такое предположение: «В подвалах дома Пашкова Воланд замечен не был. С текстами Герберта вроде бы мы его тоже не видим. Неужели соврал? — Нет. Лишаем слова Воланда чрезмерной конкретики и получаем: Воланда интересует некая рукопись, написанная неким Фаустом и хранящаяся в некоем московском подвале. А в таком виде это ложь? — Нет!». [2] Это сообщение подтверждается справкой из библиотеки имени В.И. Ленина о том, что указанных рукописей они не хранят и никто их никогда у них не запрашивал. [3] Аствацатуров не смог никак прокомментировать эти загадочные слова д. Кураева. Ничего не дал и запрос, направленный следствием на кафедру научного атеизма Московского университета. Все, что можно извлечь из подготовленной там справки: «Воланд» есть традиционное христианское название для так называемого «сатаны, отца лжи». Почему же д. Кураев считал, что в данном случае человек, назвавшийся Воландом, сообщил свою настоящую причину пребывания в Москве? Вероятно, он основывался на полученных от Понырева сведениях о товарище О.Э. Мастере, писателе, который был найден мертвым после визита т.н. «Воланда» в столицу. [4] Указанный Мастер действительно проживал в подвальном помещении и действительно опубликовал фрагмент своего романа о Понтии Пилате в советском литературном журнале. Известно, что Воланд проявлял особый интерес к предмету этого романа.

Как соотносятся высказывания (1) «я прибыл, чтобы изучить рукописи чернокнижника Герберта Аврилакского, обнаруженные в библиотеке имени В.И. Ленина» и (2) «я прибыл, чтобы изучить рукопись, написанную неким Фаустом в некоем московском подвале»? Насколько можно понять, речь идет о том, что термин «чернокнижник Герберт Аврилакский» ставится в соответствие термину «Фауст», термин «библиотека имени В.И. Ленина» ставится в соответствие термину «московский подвал». Но разве это позволяет сказать, что истинность второго суждения переходит к первому суждению?

Согласно классической концепции истины, истинны те суждения, которые находятся в соответствии с объективной действительностью. Эта концепция лежит в основании марксистско-ленинской теории истины, значит, она в основе своей верна. Но до какой степени суждение должно соответствовать действительности? Кажется, что суждение (1) само по себе действительности не соответствует: Воланд, если он действительно его вынес, врал, вводил в заблуждение своих собеседников. А суждение (2), насколько известно следствию и если принять, что «Фауст» значит «Мастер», истинно. Проблема, которая видится тут следствию, состоит в том, что ложь сама по себе не есть полное отрицание действительности. Она тоже соответствует ей, просто неверно отражает ее. Скажем, суждение «Ленин был вождем Октябрьского переворота» ложно, так как Ленин был вождем не переворота, а революции. Но все же термины «Ленин», «вождь» и «переворот» имеют объективные корреляты в действительности, просто в ней они связаны не так, как в этом суждении.

Быть может, д. Кураев рассуждал так. Известно, что имена, обозначающие некоторый объект в мире, могут соединяться и образовывать более сложные, составные имена. Эти более сложные имена также должны обозначать некий объект, т.е. иметь денотат. И при этом «денотат составного имени не меняется, если одно из входящих в него составляющих имен заменить другим с тем же, что у заменяемого, денотатом». [5] Это, как считается, работает и в случае предложений. Если в предложении заменить один нелогический термин другим, обозначающим тот же самый объект, значение предложения не изменится, хотя при этом может измениться его смысл: так, например, термин «Ленин» всегда можно заменить на термин «В.И. Ульянов». Но если это так, то от высказывания «Ленин — автор “Материализма и эмпириокритицизма”» можно перейти к высказыванию «В.И. Ульянов — это человек, написавший все 55 томов ПСС В.И. Ленина». Аналогично, это высказывание, сохраняя все денотаты, легко переформулировать в если и не синонимичное, то по крайней мере близкое ему высказывание: «число, равное числу томов ПСС В.И. Ленина, написанных В.И. Ульяновым, равно 55». Отсюда тем же методом переходим к выражению «число, равное году от Р.Х., в который апостол Павел написал 1-е послание к Коринфянам, есть 55». Поскольку имена в предложении, которым мы закончили, имеют те же самые денотаты, что и имена в предложении, с которого мы начали, у самих этих двух предложений тоже должен быть один и тот же денотат. Однако очевидно, что эти предложения совершенно друг на друга не похожи. Что же в таком случае может служить их общим денотатом и что их объединяет? Единственной их общей чертой является тот факт, что оба они истинны. Соответственно, можно утверждать, что предложения по своей природе суть сложные, составные имена, чьими значениями выступает их истинностная оценка, [6] и что все истинные предложения обозначают одно и то же — Истину, а все ложные предложения — Ложь. [7] И в таком случае дело не в том, что истинность суждения (2) некоторым образом переносится на суждение (1), но в том, что замена простых имен в суждении (2) на имена с тождественным денотатом, которая трансформирует его в суждение (1), не изменяет денотат суждения как составного имени; и если суждение (2) обозначает Истину, то и суждение (1) будет обозначать ее же. Если это рассуждение верно, то Истина и Ложь суть значения составных имен, которыми являются предложения, т.е. некие объекты, на которые эти предложения указывают. Но что это за объекты? Это явно не могут быть конкретные объекты в мире, как дерево или Ленин; значит, это должны быть абстрактные объекты. Тогда природа истины — быть абстрактным объектом.

Подобного рода рассуждения можно встретить в современной логике. Они восходят к идеям Г. Фреге, который вводит понятие «Wahrheitswert», заимствуя его у В. Виндельбанда. [8] Такое понимание истины непросто понять. Что это за абстрактные объекты? Приходит в голову аналогия с направлением линии: мы не можем дать его строгую дефиницию, но можем увидеть, что у двух линий оно одинаково, если они параллельны (и что у двух предложений одно истинностное значение, если они оба истинны или оба ложны). [9] Но почему мы вообще должны хоть на миг допускать, что ответ «Истина есть абстрактный объект» — это хорошее решение нашей проблемы? Да, оно объясняет, почему суждения (1) «я прибыл, чтобы изучить рукописи чернокнижника Герберта Аврилакского, обнаруженные в библиотеке имени В.И. Ленина» и (2) «я прибыл, чтобы изучить рукопись, написанную неким Фаустом в некоем московском подвале» могут быть эквивалентны. Но оно все же не объясняет того, откуда нам известно, что (2) истинно, и чтó эквиваленция (1) и (2) дает нам в познавательном плане. Но — главное! — неясно, насколько тезис «истина есть объект» является окончательным в деле объяснения природы истины. Противоречит ли он другим тезисам, например, тезисам «истина есть свойство», «истина есть отношение» или «истина есть событие»? Если Истина — это объект, к которому относятся все предложения определенного типа, то не будет ли верно сказать, что все эти предложения разделяют свойство «быть истинным»? Тогда что более важно для разговора об истине как таковой: Истина как объект, истина как свойство или истина как отношение причастности того, что истинно, к Истине? Это важный вопрос для следствия. [10]

Считаю необходимым в рамках следственных мероприятий подготовить проект запроса на философский факультет МГУ с целью истребования служебного пояснения по вопросу о сущности истины. Эта институция предпочитается в данном случае Институту философии АН СССР на том основании, что в последнем работает один из фигурантов дела, Понырев.

§1. Постановка проблемы

должно остерегаться разнузданной словесной мистики [11]
Мартини [12] Хайдеггер

§1.1. Пять вопросов об истине

О чем говорят, когда говорят об истине? К этому вопросу можно подойти с разных сторон.

I. В нашей речи встречаются выражения вроде «истинный друг», [13] «истинный хозяин», [14] «истинный материалист». [15] В данном случае слово «истинный» синонимично словам «настоящий», «подлинный», «всамделишный», и проблема, которую тут может исследовать философ, состоит в том, как отличить подлинное от поддельного. [16]

II. Другой смысл слова «истина» и производных от него терминов мы встречаем в выражениях «истинное суждение», «истинное убеждение», «истинная теория» и т.п. Тут «истина», как кажется, мыслится как некоторая черта определенного рода сущего (суждения, убеждения, теории и т.п.), черта, имеющая фундаментальное значение для нашего познания. Какова природа этой черты, какой категорией ее было бы правильно описать? Является ли она субстанцией, свойством или отношением? Существует ли она вообще как что-то особенное или представляет чисто языковой феномен вроде союзов и предлогов? Что значит предикат «быть истинным»? Является ли он содержательным, т.е. добавляющим нечто существенное к своему субъекту, или он является чисто логическим предикатом, который ничего существенного не обозначает? Иными словами: какова природа истины? Изучением этой проблемы занимается метафизика истины. Рассуждения, которые читатель найдет в настоящих заметках, относятся именно к этой дисциплине. Исследование метафизики истины нужно строго отличать от других вопросов, которые можно задавать об этом предмете.

III. Что является истинным? На этот вопрос отвечают перечислением истин. [17]

Значение слова «истина», как оно употреблено в конце предыдущего предложения, должно быть тщательно разведено с его иными значениями. С одной стороны, абстрактное существительное «истина» обозначает либо свойство «бытия истинным», или «истинности», либо понятие об этом свойстве [18] — и именно этот смысл интересует философа прежде всего. С другой стороны, оно может быть употреблено в исчисляемом смысле применительно к предложениям, высказываниям, убеждениям и т.д., которые обладают указанным свойством, — «ряд этих истин...», «та прописная истина, что...» — или даже совокупности этих высказываний: «мы никогда не узнаем всей истины на этот счет».

IV. Каков критерий истины? Как узнать, является ли нечто истинным? Есть ли тест, который может отделить истину от лжи? Это эпистемический, а не метафизический вопрос. [19]

V. Особое место в исследованиях истины занимает так называемая критика истины, которая может быть рассмотрена как эпистемология, но лежащая не в позитивистской традиции, а в традиции критики культуры.

Если метафизик (II) задается вопросом о том, что есть истина, а эпистемолог (IV) — о том, как познать истину, критический философ спрашивает: как произведена эта истина, по какому праву и каковы будут последствия этого действия? Фуко писал, что в изучении истины можно выделить два аспекта: «первый аспект — определение того, как можно удостовериться в истинности утверждения, — кладет начало великой традиции западной философии, которую я бы назвал «аналитикой истины». Второй аспект — вопрос важности высказывания истины, знания о том, ктó способен высказывать истину, и понимания целей высказывания истины, — лежит у истоков того, что мы могли бы назвать «критической традицией Запада». [20] Фуко интересуется вторым способом вопрошания. Критика истины, описывающая т.н. «игры истины» [21] как социальные конструкции, не должна, однако, пониматься как отрицание ее существования. Каждая игра истины относительно своего объекта описания является именно описанием, а не конструкцией, но все же сами правила дискурсивной игры исторически изменчивы и являются потому конструкцией относительно других описаний. [22] Но это означает не то, что истин нет, а то, что они, как существующие, существуют относительно описаний, в социальном окружении. Таким образом, критическая философия тем и отличается от истории идей, что ее адептов интересует, почему некоторые явления (истина, образование, сексуальность и т.п.), которые уже были в обществе, начинают вдруг замечаться и проблематизироватьcя и почему эти проблематизации изменяются с течением времени. Такая постановка вопроса частично наследует Канту, так как в ее рамках вопрошают об условиях возможности той или иной конкретной проблематизации, а частично продолжает идеи Ницше, который задавался вопросом о том, почему мы ценим истину. [23]

§1.2. Зачем вообще нужны теории истины?

Исследования по метафизике истины интересны сами по себе. Но они также тесно связаны с другими философскими вопросами. Каждая такая связь требует специального обсуждения, так что здесь мы вынуждены ограничиться простым перечислением, которое приводится в надежде показать возможные мотивации научного интереса в области исследований истины.

(А) Метафизика: реализм и антиреализм, идеализм и материализм. Сегодня противостояние между этими позициями часто идет по линии не столько того, какой тип существования, духовный или материальный, является онтологически первичным, сколько того, как работает понятие истина. [24] Так, если бы суждения могли быть только истинными или ложными (т.е. если был бы верен закон исключенного третьего), это было бы хорошим доводом в пользу реализма (подробнее см. §2.3.4).

(В) Философия сознания: онтологический статус ментального содержания зависит от того, является ли истина отношением между убеждениями и миром. Положительный ответ на этот вопрос, особенного в случае, если это отношение — тождество, склоняет нас к экстернализму в отношении содержания ментальных понятий и актов, в случае же отрицательного ответа интернализм кажется более верной позицией. [25]

(С) Философия языка, логика и семантика. (С1) От природы истины зависит природа отрицания, важного феномена нашего языка. (С2) Современная логика в своих методах и в своей онтологии сильно зависит от понятия «истинностного значения». [26] (С3) Условно-истинностная концепция значения является одной из ведущих исследовательских программ в философии языка. [27] (С4) Истина лежит в основе многих семантических парадоксов, в частности парадокса Лжеца. [28]

(D) Этика и эстетика: одной из фундаментальных проблем этих дисциплин является познавательный статус исследуемых ими оценочных суждений. Если истина такова, что она может быть применена к оценочным суждениям, то это является доводом в пользу когнитивизма в этике и эстетике; если истина не такова, то это довод в пользу нонкогнитивизма [29] — теории, согласно которой ценностные высказывания не имеют «когнитивного» содержания, т.е. служат не познавательной, а некоторой иной функции, например выражению эмоций [30] (ср. §2.1.7.1).

§1.3. Исторический контекст и основные теории истины

Хотя «истина» (ἀληθείη) встречается уже в эпической поэзии Гомера, философское осмысление она, видимо, впервые получает у Парменида: именно на пути Истины, «что [нечто] есть и что невозможно не быть», [31] он находит бытие. [32] Менее спекулятивный взгляд на истину мы находим у софиста, чье имя история для нас не сохранила, в отличие от его «Двояких речей», датирующихся около 400 г. до н.э. В этом тексте сказано, что «когда речь высказана, если как говорится в речи, так и обстоит дело, то речь истинна; а если дело так не обстоит, то она ложна». [33] Эта интуиция позднее будет развита Аристотелем (см. §2.1). Древность также знала и трактаты с названием «Об истине», но они были посвящены скорее вопросу (I), в частности его социально-политическому аспекту, что мы находим, например, у Антифонта. [34]

В Средние века Фома Аквинский определяет истину как соответствие вещи и разума. [35] Такое нейтральное определение вводится для того, чтобы предусмотреть возможность понимания истины одновременно и в смысле (I), и в смысле (II). Иными словами, он считает, что истинными с равным основанием могут называться как вещи и люди (например, «истинный коммунист»), так и мысли: мысль является истинной, поскольку она соответствует реальности, тогда как вещь или человек являются истинными, поскольку они соответствуют мысли. В последнем случае под мыслью, однако, понимается прежде всего мысль Бога: истинный коммунист истинен настолько, насколько он соответствует божественному понятию о том, каким должен быть коммунист.

В принципе в том же духе, т.е. перебиваясь с корреспондентских интуиций на общие размышления, рассуждали об истине в Новое время. Примером тут может служить эссе Ф. Бэкона «Об истине», в котором он рассуждает о том, что «истина есть обнаженный и открытый дневной свет, при котором маски, представления и торжества мира выглядят вполовину менее величественными и утонченными, чем при свете свечей», и хотя этот свет, свет разума, есть первое создание Бога, люди все же не хотят принять его в силу «естественной, хотя и порочной любви ко лжи, как таковой». [36] С имевшим божественное происхождение естественным светом разума, то есть со способностью усматривать очевидное, связывает истину и Декарт. Это решение касается критерия истины (IV). Ясно, что делает возможным ее познание: дарованная Богом способность человеческого ума. При этом о том, как именно Декарт представляет себе сущность истины, ее онтологию, можно только гадать. [37] Спиноза, Гассенди, Локк и Лейбниц в целом подтверждают свое согласие с процитированной выше идеей анонимного софиста. [38] У Джонатана Эдвардса мы встречаем развитие локковского определения в идеалистическом ключе, которое, однако, в целом остается верным духу корреспонденции. [39]

Кант, не опровергая классическую концепцию истины, полагает, что «соответствие знания с его предметом» есть лишь номинальное определение. В поиске же всеобщего критерия истины для всякого знания смысла нет: такой критерий должен был бы быть дан лишь в отвлечении от конкретных предметов познания, т.е. формально, а между тем конкретная истина всегда имеет отношение к содержанию. Поэтому дискуссии о таком критерии описываются Кантом так: «один доит козла, а другой держит под ним решето». [40] О метафизике истины это рассуждение ничего не говорит.

Настоящей арией истины может быть названо системостроительство Гегеля. Он обращает внимание на непоследовательность в рассуждении Канта. Соглашаясь с номинальным определением истины, тот утверждает, что истина есть соответствие. А критикуя возможность всеобщего критерия истины, он говорит о том, что истина связана с содержанием знания. Это рассуждение Гегеля можно толковать уже не в духе эпистемологии, а как метафизику истины: он указывает на то, что в первом случае истиной называется отношение познания к своему предмету, а во втором — содержание познания, т.е. его свойство. [41] Выходом из этой путаницы Гегель полагает нахождение собственного, внутреннего, а не внешнего, содержания у самой формы знания, [42] т.е. истина в итоге оказывается тождеством реальности и понятия. [43] (Под «реальностью» тут имеется в виду не чувственная якобы реальность, а внутренняя понятию идеальность.) [44] Такая истина возможна только одна, и это Абсолютная идея, которая «есть бытие, непреходящая жизнь, знающая себя истина и вся истина». [45] Ему же, Гегелю, принадлежит честь сделать обыкновением философской речи такие конструкции, как «истина того-то», «истина сего-то», в значении «суть того-то», «суть, сущность или природа чего-то». Через это вопрос о природе истины был поставлен как метафизический: то, что Гегель называет логикой, т.е. наукой об истине, [46] есть то, что мы сегодня называем онтологией, или метафизикой. [47]

Удивительно, что, хотя истина является классическим философским понятием, серьезную пролиферацию теорий истины мы наблюдаем только с разложением гегелевской школы. Продуктами этого разложения и были субъективистская теория истины, развитая в рамках индивидуализма М. Штирнера и С. Кьеркегора, [48] теория истины как практики, предложенная в рамках диалектического материализма Маркса, эпистемическая теория истины, возникшая на основе прагматизма Пирса, критика истины в творчестве Ницше и теория тождества Брэдли, бывшая составной частью его идеализма. Субъективистская концепция истины, как и подход Ницше, касается не столько вопроса (II), сколько вопроса (V), что особенно интересно в свете возможного влияния Кьеркегора на Фуко. [49] Марксистская теория практики как критерия истины в своих стандартных изложениях является ответом на вопрос (IV), но может быть рассмотрена и как онтология истины. [50] Эти теории составили первый этап современных исследований истины. Им наследуют семь основных концепций метафизики истины, которые сегодня активно обсуждаются в философии: корреспондентская, или классическая (она же теория соответствия), когерентная (она же теория согласованности), эпистемическая (подвидом которой можно считать прагматическую), теория истины как тождества, дефляционитская, примитивистская и плюралистическая. [51]

Сегодня почти все сторонники этих семи подходов полагают, что принципиально верной и способной составлять по крайней мере часть метафизики истины (а возможно, и прямо ее основу) является семантическая теория истины А. Тарского.

§1.4. Семантическая концепция истины А. Тарского

Задачей Тарского было создать такую формально корректную (т.е. хорошо структурированную и удовлетворительно описанную) теорию истины, которая учитывала бы все правильные (т.е. согласующиеся с реальным способом его использования в языке) употребления предиката «быть истинным» (т.е. была бы материально адекватна), при этом не постулируя существование нефизических сущностей [52] и не впадая в парадокс Лжеца. Базовыми носителями истины (см. §1.7) у Тарского выступают предложения. Возьмем для примера предложение «снег бел». Когда оно истинно? Очевидно, что предложение «снег бел» истинно, если и только если снег бел. В правой части этой эквиваленции стоит само предложение, а в левой, в кавычках, — имя этого предложения. Если обобщить эту процедуру, то мы получим такую формулу:

(Т) X истинно, если и только если р.

Эта формула называется Т-схемой, [53] или тезисом эквиваленции. Важно отметить, что это не предложение, а схема предложения. Неверно было бы также понимать ее в качестве определения истины, как это нередко делается. Эквиваленцию вида (Т), получаемую подстановкой на место р конкретного предложения, а на место Х имени этого предложения, можно считать частичным определением истины; в качестве же общего определения истины можно рассматривать только полную конъюнкцию всех случаев таких «частичных» определений.

Теперь поговорим о понятии «формальная корректность». Теория формально корректна, когда ее язык является точно определенным. Это значит, что в таком языке однозначно определяется класс слов, которые считаются значимыми, в этом классе указываются все примитивные термины, т.е. слова, которые мы будем использовать без определения, даются правила определения для введения в язык новых определяемых терминов, даются критерии для определения класса предложений, формулируются условия, при которых предложение может утверждаться, указываются все аксиомы и даются правила вывода для новых утверждаемых предложений из тех, утверждаемость которых уже была установлена. При этом язык, который нас интересует при построении теории истины, не должен быть семантически замкнут, т.е. не должен сам содержать помимо предложений имена этих предложений. Семантически замкнутым является естественный язык, поэтому в нем и возникает парадокс Лжеца. Это соображение приводит Тарского к различию между объектным языком, т.е. совокупностью предложений, к которым применима истина, и метаязыком, в терминах которого и формулируется определение истины. Метаязык должен быть существенно богаче объектного языка: то есть должно быть неверно, что любому термину метаязыка можно сопоставить вполне определенный термин объектного языка, иначе парадокс Лжеца мог бы возникнуть на новом уровне.

Само различие между объектным языком и метаязыком мотивировано не только желанием разрешить парадокс Лжеца, но и наблюдением о связи имен и объектов. Свойства в нашем языке всегда приписываются объектам не непосредственно, а через их имена: так, в предложении «Сталин — красный» мы приписываем свойство «быть красным» Сталину через его имя, «Сталин». В самом деле, если бы мы хотели приписать в нашей речи Сталину свойство «быть красным» непосредственно, минуя имя «Сталин», нам пришлось бы поместить в наше предложение вместо его имени самого Сталина — очевидно, что язык так не работает. Поэтому и для того, чтобы сказать о предложении, что оно истинно (т.е. приписать ему свойство «быть истинным»), мы должны использовать имя этого предложения, а не его само: само описываемое предложение, будучи объектом иного порядка, нежели его имя, может напрямую фигурировать в нашем высказывании не более, чем физический Сталин может фигурировать в нашем языке наряду с именами и глаголами.

Таким образом, р в Т-схеме обозначает предложение объектного языка, а Х — это имя этого предложения, принадлежащее метаязыку, как и сама Т-схема и результаты подстановок в нее. Это означает, что не существует истины вообще, истина существует всегда относительно некоторого языка.

Важно отметить, что в таком виде Т-схема связывает два лингвистических объекта — предложение (снег бел) и его имя («снег бел»), а не лингвистический объект («снег бел») и положение дел во внешнем мире (факт белизны снега). Это означает, что истинность одного языкового объекта, строго говоря, ставится в ней в зависимость не от мира, а от другого языкового объекта. Т-схема как бы оставляет нас в пределах языка и не предполагает прямого «выхода» в мир, в то время как обычное употребление предиката «быть истинным» часто явно этот выход имеет в виду. В концепции Тарского связь Т-схемы и мира осуществляет выполнимость — отношение между произвольными объектами и пропозициональными [54] функциями (ПФ). ПФ — это, грубо говоря, выражения, структура которых аналогична предложениям, но которые содержат свободные переменные: «х — белый», «у — черный» и т.д. Выполнимость — это, грубо говоря, отношение между ПФ — например, «х бел», — и тем объектом, при подстановке которого на место свободной переменной в этой функции мы получим истинное предложение: например, объект «снег» выполняет ПФ «х бел». Ни первое, ни особенно второе определение не являются точными — поскольку Тарский намерен определить истину с помощью терминов «ПФ» и «выполнимость», сами эти термины нельзя определять через истину (как мы сделали в случае «выполнимости»). Поэтому оба этих термина Тарский определяет рекурсивно: на первом шаге перечисляя все без исключения допустимые в конструируемом языке случаи, к которым приложим этот термин, а на втором — задавая правила того, как из этих простых случаев конструировать более сложные. Иными словами, мы сначала определяем ПФ, перечисляя все простейшие случаи таких функций в нашем языке («х — бел», «у — зелен», «z — красен» и т.д.), [55] а затем перечисляем для каждой упомянутой функции все объекты, которые ее выполняют (для «х — бел»: снег, лед, Колчак и т.д.) [56] и тем самым определяем выполнимость.

На основании этого Тарский дает определение истины и ложности для предложений, т.е. таких пропозициональных функций, где на место переменных уже подставлены имена: некоторое предложение истинно, если выполнимо всеми объектами, в противном случае оно ложно. Это определение нуждается в пояснении. Предложение «снег бел» выполняется всеми нашими объектами не потому, что «х бел» выполняется всеми нашими объектами (эта функция не выполняется травой или Махно), а потому, что «снег» встречается в нашем заранее заданном на этапе определения «выполнимости» списке объектов, выполняющих «х бел». «Выполняется всеми объектами» здесь фактически значит «выполняется вне зависимости от объектов»: «снег бел» — это пропозициональная функция, которая уже выполняется вне зависимости от того, имена каких объектов мы попытаемся в нее подставить, — просто потому, что в ней уже нет переменных, а поименованный в ней объект встречается в списке выполняющих функцию «х бел». Предложение же «трава бела» не выполняется никакими объектами, потому что в нашем списке объектов, выполняющих функцию «х бел», нет травы.

Семантическое исследование Тарского стало основанием многих современных метафизических теорий истины, прежде всего некоторых вариантов корреспондентского подхода, почти всех версий дефляционизма, а также примитивизма и плюрализма. Потому неудивительно, что существует множество интерпретаций полученных Тарским результатов. Некоторые из них сближают Тарского с корреспондентской теорией. [57] В этом случае его вклад в развитие этой теории по отношению к уровню, обнаруживаемому у Аристотеля, можно суммировать так: (1) Тарский дает непротиворечивую интерпретацию корреспондентского подхода, которая избегает парадокса Лжеца; (2) его теория раскрывает семантическую роль предиката истины; (3) его теория устанавливает отношения между объектным языком и метаязыком, а не напрямую между словами и вещами. Другие толкуют его в дефляционистском духе. [58] Это вполне разумное толкование, если не отождествлять дефляционизм с теорией избыточности (см. §2.5.1), но сам Тарский выражал намерение развить именно корреспондентский подход.

§1.5. Терминологическая проблема?

Тарский считал, что его теория не единственно возможный подход к истине. Из-за неопределенности естественного языка одно и то же слово может иметь разные смыслы, и дело семантики — строго их определять и изолировать друг от друга. [59] Но тогда, быть может, упомянутые семь теорий просто фиксируют разные смыслы слова «истина»? Например, кажется, что истинность суждений о физическом мире неплохо описывает корреспондентская теория, а для описания истинности этических суждений скорее подойдет когерентная теория. Может быть, тогда нет смысла искать единой теории истины, и для каждой дискурсивной области мы могли бы просто постулировать свое собственное свойство истинности?

Такой подход сталкивается с т.н. проблемой смешанных высказываний. Представьте себе такое рассуждение: «Если шпионы вредят Родине, то гражданин должен быть бдителен. Шпионы вредят Родине. Следовательно, гражданин должен быть бдителен». Первая посылка — смешанное суждение, так как, по предположению, его антицедент есть суждение, чья истинность имеет корреспондентскую природу, а консеквент — когерентную. Если истинность антецедента и истинность консеквента — это два разных свойства, то не существует способа установить условия истинности импликации в целом. А это делает невозможным рассуждение, первой посылкой которого эта импликация является. Однако это рассуждение (как и многие другие, подобные ему) очевидно является не только возможным, но и совершенно верным! Таким образом, метафизика истины не может быть сведена к чисто терминологической проблеме. [60]

§1.6. Проблема выбора

На консультации по проблемам онтологии и теории познания для следователей КГБ, проведенной доцентом философского факультета МГУ В.Ю. Кузнецовым, последним был поставлен вопрос: нам нужно выбрать истинную теорию истины, но как это возможно, если мы еще не знаем, что есть истина? К чему этот вопрос был поставлен Кузнецовым, персонально мною, а также присутствующим там же личным составом было не понято. Но вопрос интересен сам по себе. Я считаю, что решать его нужно, опираясь на научный метод. Прежде чем выбрать истинную теорию истины, мы должны зафиксировать то, что мы об истине знаем и без всяких теорий. А мы без сомнения что-то о ней знаем, так как постоянно используем этот термин, и часто успешно. Вот что кажется мне несомненным: (1) желая проверить, существует ли факт, чье существование указывается содержанием эмпирического суждения, мы должны обратиться к реальности, которая находится за пределом этого суждения, и это обращение можно правильно описать как установление истинности этого суждения; (2) мы можем выносить истинные суждения случайно, без того чтобы знать об их истинности; (3) после установления того, что некоторый факт имел место, нет необходимости дополнительно доказывать, что описывающее этот факт суждение истинно; (4) если установлено, что некоторое суждение истинно, то это может быть основанием для согласия с этим суждением. [61]

Этот список может быть уточнен, дополнен или изменен в ходе исследований того, что такое истина. Теория, которая не адекватна этому минимальному набору черт истины, — неистинная теория истины. Выбор между теориями, равно адекватными этому набору, будет осуществлен на основании оправданного расширения этого набора. Кроме того, ряд теорий истины может быть исключен как заведомо ложный на основании внутренней непоследовательности или заведомого допущения невозможного.

§1.7. Носители истины

Понятие «носитель истины» (truth-bearer) имеет свои корни в религиозном контексте, в котором иногда различают саму истину и то, откуда истина приходит. [62] В современной философии этим термином обозначают те сущности, которые обладают свойством «быть истинным». В литературе мы встречаем несколько кандидатов на роль носителей истины. Перечислим их, подмечая их сильные и слабые стороны. Отметим, что терминология в этой области сложно устроена, значения не закреплены однозначно, разные авторы используют одни и те же термины по-разному: иногда это связано с различием в теоретических позициях, иногда — нет. Дополнительную трудность создает то, что основная научная литература по носителям истины написана по-английски, установить взаимно однозначное соответствие между логико-лингвистическим словарем английского и русского языков не представляется возможным за пределом узко формальных исследований.

(1) Пропозиции, они же суждения, они же высказывания (propositions, judgments), или мыслимое (thinkable), или мысли (thoughts). Это логические, нелингвистические и непсихологические сущности, которые могут быть описаны через четыре основных свойства: а) они являются смыслами (или содержаниями) предложений; б) они являются субъектами т.н. пропозициональных установок (например, «я знаю, что коммунизм победит»; следовательно, существует то, что знаю; это нечто и называется пропозицией); в) они могут быть истинными и ложными в абсолютном, неизменном смысле; г) одно и то же предложение в разных контекстах может отсылать к разным пропозициям. Для многих философов, которые специально не работают с проблемой истины, разговор о пропозициях является обычным делом. [63] Вплоть до начала ХХ века ведущие логики, такие как Больцано и Фреге, считали, что их наука зависит от существования пропозиций. К середине века усилиями Тарского и Куайна пропозиции в логике стали вытесняться предложениями; впрочем, то, что современной логике удобнее работать с предложениями, не значит, что пропозиций не существует и что не они выступают подлинными носителями истины. [64] Сегодня «чистые» логики если вообще обсуждают носители истины, в основном предпочитают рассматривать в качестве таковых предложения. [65] Тогда как в метафизике истины маятник в последние два десятилетия качнулся в обратную сторону, и наиболее распространенными носителями истины вновь являются пропозиции. Однако постулирование существования пропозиций и назначение их носителями истины сталкивается со следующей проблемой: неясно, каков онтологический статус пропозиций и их место в мире. Некоторые теории пропозиций фактически отождествляют их с положениями дел (т.н. «расселианские» пропозиции), другие требуют реализма относительно универсалий и/или абстрактных объектов, третьи помещают их внутрь акта высказывания, пытаясь таким образом предложить третий путь, альтернативный как номинализму, так и реализму; [66] существуют и иные трактовки природы пропозиций. [67]

(2) Предложения — лингвистические сущности. Выделяют предложения-токены и предложения-типы. Многие современные специалисты в области исследования истины считают именно предложения-токены носителями истины. Хорошую репутацию предложениям обеспечил Тарский, который сам, однако, предпочитал предложения-типы. [68] Предложения хороши тем, что их можно бихевиорально обнаружить, у них есть строгая грамматическая структура, мы знаем, из чего состоят предложения. Предложения плохи тем, что могут включать в себя индексалы («я», «здесь», «вчера»), в результате чего их истинностное значение будет меняться в зависимости от контекста и ситуации и истина утратит стабильность. Эта проблема решается введением т.н. вечных предложений, т.е. предложений, в которых все индексальные термины переведены в термины, поддающиеся четкому референциальному определению. [69] Другой проблемой предложений является контринтуитивность выбора их в качестве носителей истины: как простые звуки или значки на бумаге могут переносить такое семантическое свойство, как истина, сами по себе, без посредства абстракций или мыслей?

(3) Положения, высказывания, утверждения (statements, utterances, assertions) — это события, в которых посредством предложений (или чего-то аналогичного, например жеста) выражается какой-то смысл (или пропозиция). Одно предложение может быть использовано для вынесения разных утверждений. Возможно и обратное: разные предложения для одного утверждения. Онтологический статус этих сущностей ясен, и если мы принимаем, что в мире существуют события, то нет проблем принять существование положений или высказываний. Однако события привязаны к конкретному моменту времени, и если мы выбираем их в качестве носителей истины, то делаем проблематичным разговор о знании, чей истинностный статус prima facie неизменен и не зависит от акта высказывания: например, законов логики или математики.

(4) Убеждения, или верования (beliefs), идеи, образы, ментальные состояния в целом. Вырожденным случаем этого варианта являются «имена», которые упоминает Джеймс. [70] Достоинством этого кандидата является его непосредственная доступность нам во внутреннем опыте. Одним его недостатком является непроясненный онтологический статус ментального. Другим — то, что не очевидно, как ментальные образы могут быть истинными или ложными, скорее они, как образы, могут быть более или менее точными. [71]

Какова роль изучения носителей истины в общем контексте исследований истины? В целом она, вероятно, невелика. Почти каждый из кандидатов может быть выбран в качестве носителя истины, вопрос лишь в том, какой из носителей является первичным и фундаментальным, причем ничто не указывает на то, что такой носитель может быть только один. Это, однако, не значит, что этот вопрос совсем не важен. Необходимо осознавать, что выбор того или иного носителя истины может заранее исключить какие-то метафизики истины. Так, если пропозиции суть абстракции, и абстракции не могут вступать в каузальные отношения, то это исключает те версии корреспондентской теории, которые требуют каузальной связи того, что делает нечто истинным, и носителя истины. [72] Выбор высказываний и убеждений в качестве носителей истины означает, что после гибели человечества и в случае невозникновения новой формы разумной жизни в мире не останется истины. Это значит, что через несколько миллионов лет «2=2» не будет истинным выражением. Многие могут посчитать это абсурдным следствием. Выбор любого кандидата, кроме пропозиций, исключает теорию тождества (см. §2.4).

§2. Что есть истина?

What is truth? said jesting Pilate, and would
not stay for an answer.
«Of Truth» Francis Bacon

§2.0. Вступление

Когда же настало утро, первосвященник Иосиф Каиафа, его родственник Анна, книжники и их прислужники отвели связанного Иешуа в преторию. Там они ввели его в небольшой зал, где на троне сидел пятый прокуратор Иудеи, всадник Золотое Копье, Понтийский Пилат. По левую руку от него сидел секретарь, по правую — начальник тайной службы Афраний и кентурион, командующий особой кентурией, Марк, прозванный Крысобоем. На ложе за троном полулежала Прокла, супруга Пилатова. Встал Иешуа перед правителем.

Пилат (стараясь не смотреть на бородавку на лице и родимое пятно на голове арестанта): Ты царь Иудейский?

Иешуа: Так говорят.

Каиафа: Ты сам так назывался, народ видел! Юлий Латинянин, писарь, свидетельствует о разбойных делах его! И даже бездомный Йоханан Поныр говорит то же: злой он и разбойные дела чинит власти и простому люду!

Пилат: А тебе, Каиафа, завидно стало? Разве не было у народа вашего царей ранее? Может быть, ты сам хотел бы принять этот титул?

Анна: Что говоришь ты, прокуратор? Как бы не услышал нас кто-нибудь, игемон! Ведь твою шутку народ может и не понять.

Пилат: Ну будет тебе, Анна, все воспринимать так серьезно. Впрочем, возможно, ты и прав, дело нужно завершить быстрее. (Обращается к Иешуа.) Если же ты, Иешуа Назорей, странствующий врач и философ, называл себя царем Иудейским, то это бунт против власти кесаря! Если царь — ты, то налоги, выходит, платить народ иудейский должен тебе, а не ему!

Иешуа: Если.

Пилат: Ты отрицаешь обвинения уважаемых людей, писарей, священников и книжников?

Иешуа: Пусть скажут, что я никогда не просил так меня называть! Я учил всегда в синагоге, где народ, по обычаю, сходится, никогда не учил тайно. Пусть спросят тех людей, кто меня слушал, они скажут, что я никогда не назывался царем. Что до слов писаря Латинянина, то внимания на них обращать мудрый человек не будет.

Анна (дает пощечину Иешуа): Так ты, пес, отвечаешь прокуратору?!

Пилат: Если он сказал худо, то скажи, что худо. Если же сказал хорошо, то чего ты, Анна, бьешь его? Какое же зло он совершил?

Анна и Каиафа: Он развращал молодежь иудейскую сладкими речами!

Пилат: Это не преступление. (Обращается к Иешуа.) Скажи, за что народ твой так тебя ненавидит?

Иешуа: Что ты, игемон, народ любит меня и покупает еду, когда я очень нуждаюсь, укрывает в домах своих от бурь, когда те гостят в Иерусалиме. Это лишь люди первосвященника меня невзлюбили, потому что я свидетельствую об истине, что мне делать легко и приятно. А им доставляет неудовольствие, когда звучит истина и все ее узнают.

Пилат: Что есть истина, безумец? Наши юристы и греческие любители мудрости уже отчаялись ответить на этот вопрос. Как же это ты, ты, иудейский бродяга, смущаешь народ, рассказывая про истину, о которой ты не имеешь представления?!

Иешуа: Но я имею о ней представление, игемон!

Пилат: Что ж, позабавь меня! Если речи твои будут забавны, то велю послать запись твоего допроса в Александрию, в дома Филона [73] и Аммония. Может, хотя бы в философии ты произведешь переворот, а, бродяга?

§2.1. Корреспондентская теория

Иешуа: Изволь. Истина прежде всего в том, что у слуги твоего Афрания болит голова, и болит так лихо, как лихи были вчерашние его увеселения. (Секретарь перестает писать, замерев от ужаса. Анна, Каиафа и другие книжники подаются к Иешуа, но замирают, поймав на себе взгляд Пилата.) Почему это так, и что тут, собственно, истинного в этой истине? Истинно прежде всего убеждение в том, что у Афрания болит голова. Почему оно истинно? Потому что у него болит голова. Нечто является истинным или ложным, так как другое, отличное от него, нечто делает его истинным или ложным. Причем важно, что первое нечто должно быть выражено убеждением, а второе нечто быть чем-то, что существует в мире, который находится вне этого убеждения. Вот мое учение об истине, игемон.

Пилат: Скажи, ты учился где-то сам? Отвечай обдуманно, потому что если я усмотрю в речах твоих преступление, то участь твоя будет незавидна. А ты, как видно, не очень отличаешь простодушие от дерзости.

Иешуа: Я, игемон, читал книги, и только.

§2.1.1. Зерна, которые бросил Аристотель

Пилат: То, что ты говоришь об истине, мне известно со слов одного давно уже мертвого грека из Стагиры по имени Аристотель. Он несколько раз определял истину, причем показательно, что все определения мы находим в его недавно опубликованной «Метафизике», а не в давно известном нам Органоне. Вот его определения, я помню их наизусть: (1) «говорить о сущем, что его нет, или о не-сущем, что оно есть, — значит говорить ложное; а говорить, что сущее есть и не-сущее не есть, — значит говорить истинное»; [74] (2) «истинно утверждение относительно того, что на деле связано, и отрицание того, что на деле разъединено; а ложно то, что противоречит этому разграничению»; [75] (3) «истину говорит тот, кто считает разъединенное разъединенным и связанное — связанным, а ложное — тот, кто думает обратное тому, как дело обстоит с вещами». [76] Суть этой теории можно выразить так. Не потому ты бледен, бродяга, что мы говорим, что ты бледен, а истинно, что ты бледен, потому что ты бледен. Отношение подчинения идет не от слов к миру, а от мира к словам. Положения дел в мире делают то или иное суждение о них, как ты и говоришь, истинным или ложным. По Аристотелю, мир, пожалуй, надо называть действующей причиной истинности суждений. То, что мое убеждение о том, что ты, бродяга, стоишь предо мною, является истинным, обеспечено тем фактом, что на мою чувственную способность на протяжении жизни воздействовали люди, и некоторые из них были бродягами, и потому я сформировал общее понятие о бродягах. Я также видел немало бледных вещей, и они были причиной того, что я знаю, что такое бледность. При этом форма бледности во внешнем мире и форма бледности в моих чувствах формально, но не нумерически совпадают. То же самое с формой бродяги. Поэтому истинность суждения «бродяга бледен» обеспечивается фактом соединения субъекта (бродяга) и предиката (бледность) этого суждения в реальном мире. Тут, я думаю, имеет место три действующих причины: множество бродяг, которое я имел в опыте до сих пор, множество бледных, которые я имел в опыте до сих пор, и бледный бродяга, который сейчас передо мной. Наши мудрецы называют это корреспондентской теорией и считают ее ошибочной. Этому ты, бродяга, учишь?

§2.1.2. Рассел: теория суждения как многоместного отношения

Анна: Игемон, это ложное учение! Во-первых, разве каждый человек думает о том, что «бродяга бледен», когда видит бледного бродягу? Нет! Это значит, что нет необходимой связи между мыслью о бледном бродяге и самим этим бродягой! Какая же это тогда причина, ведь причинность мыслится как нечто необходимое?! Во-вторых, эта теория предполагает, что истинные высказывания должны быть каузально связаны с миром. Но как быть со случайно истинными высказываниями? Их-то объяснить таким же образом нельзя, потому что у случайного, как говорит ваш Стагирит, нет причин и, значит, нет объяснения!

Иешуа: Я не учу, игемон, тому, как мы получаем истинное знание. Я учу лишь тому, что значит для знания быть истинным: знание должно быть адекватно реальности.

Пилат: Что же тогда связывает знание и реальность, если не причинность?

Иешуа: Их связывает убеждение, только важно понимать, что это такое. Убеждение не является двухместным отношением между умом, которому оно присуще, и объектом убеждения, то есть фактом. Если бы это было так, то у всякого убеждения был бы свой объект, даже у ложного. Например, что за объект у убеждения «в этом зале находится носорог»? Такого объекта нет. [77] Тогда что делает его ложным? Постулировать существование такого объекта как «отсутствие носорога в зале» было бы странно. Поэтому убеждение лучше понимать как многоместное отношение. То, что называют убеждением или суждением, есть не что иное, как отношение, связывающее ум с несколькими вещами, отличными от него, — объектами этого убеждения. Допустим, добрый человек Анна убежден в том, что по левую руку от игемона сидит его секретарь. Его убеждение — это отношение, которое создает сложное целое из Анны (субъекта убеждения), сидения слева от, игемона и секретаря (объектов убеждения). Это убеждение истинно, так как существует другое сложное целое, называемое фактом: секретарь сидит слева от игемона. Это последнее сложное целое также создается отношением между его объектами, но другим: отношением «сидение слева от». И так как это сложное целое составлено ровным счетом из тех предметов, которые были объектами убеждения Анны, расположенных в том же самом направлении, или порядке, то его убеждение истинно: оно истинно, потому что существует изоморфизм между структурой убеждения и структурой факта. Это убеждение было бы ложным, если бы элементы «секретарь», «сидение слева от» и «игемон» не образовывали бы в реальности того сложного целого, в котором убежден Анна. Валлиец Бертранд Рассел учит тому. [78] Я следую его учению в своих разговорах с людьми.

Пилат: Но для этого нужно, чтобы каждый термин суждения соответствовал какому-то объекту в мире. Но что значит это «соответствие», в чем оно-то состоит?

Иешуа: Мы просто знакомы с ними. Игемон знаком с бродягами, знаком с бледностью.

Пилат: А если Анна ошибается? Если он считает, что секретарь слева от меня, а это ему только так кажется? Если он за секретаря принял кого-то еще?

Иешуа: Тогда, как я уже сказал, просто нет сложного единства, в котором убежден Анна, и его суждение ложно.

Пилат: Да, но ведь он знаком и со мной, и с сидением слева, и с секретарем. Как же возможна ошибка?

Иешуа: Само знакомство непогрешимо, но выводы, которые мы делаем на его основе, могут быть ошибочны. Анна в твоем примере ложно убежден, что находится в отношении видения секретаря.

Пилат: Ты сказал, что знакомство непогрешимо. Ты сказал, что знакомство устанавливает связь между термином и тем, что он значит. Так как же возможна ошибка, как возможно, что термин «секретарь» значит что-то другое, например галлюцинацию Анны?

Иешуа: Он не значит другое. Он значит секретаря. И если секретарь — не слева от игемона, то убеждение о том ложно.

Пилат: И все же я чувствую тут какую-то трудность... Но я не буду давить на тебя в этом вопросе. Отвечай мне лучше вот на что: учишь ли ты, что каждому факту соответствуют только два убеждения, одно истинное, другое ложное, а для каждого истинного суждения существует лишь один соответствующий ему факт?

§2.1.3. Теории факторов истинности

Иешуа: Так, кажется, учил когда-то Бертранд Валлиец, но я того не говорил. Он и сам впоследствии скорректировал свое мнение по этому поводу. [79] В случае простых убеждений или высказываний об отдельных предметах это, может быть, и верно, тут мне самому это пока не до конца ясно. Но в случае, например, обобщающих высказываний это очевидно не так. Например, когда я говорю «мы с игемоном обсуждаем теорию истины», то это высказывание будет истинным, если мы обсуждаем теорию истины Аристотеля, теорию истины Рассела или еще какую-то другую: одно высказывание делают истинным разные факты. И, наоборот, один и тот же факт может делать истинными разные высказывания: тот факт, что слева от игемона сидит секретарь, делает истинными высказывания «слева от игемона сидит секретарь», «слева от игемона сидит человек», «слева от игемона сидит кто-то» и так далее. Между фактами и высказываниями в целом нет одно-однозначного соответствия. Последователи Рассела и его ученика Хлодвига Витгенштейна разработали эту теорию очень подробно. Они говорят, что для высказывания, или пропозиции, быть истинным значит иметь фактор истинности: [80] что-то в мире, что делает это высказывание, или пропозицию, истинным. Разделяющих подобное учение людей иногда зовут не теоретиками корреспонденции, но сторонниками теории факторов истинности. [81]

Пилат: Хорошо же их определение! Они поймали себя в круг: истинность у них определяется через то, что делает истинным.

Иешуа: Твоя правда, игемон. Но эти люди не стремятся дать определение истины, а только объяснить, что она такое, так, чтобы это стало понятнее.

Пилат: Я не уверен, что это с их стороны честно, но допустим. Но как они понимают факторы истинности?

Иешуа: Сказать честно, по-разному. Но обычно они считают, что фактор истинности — это факт, положение дел в мире, за счет которого соответствующая ему пропозиция становится истинной.

Пилат: Но неужели они думают, что все пропозиции на свете им удастся объяснить подобным образом? Я понимаю, что дизъюнктивные пропозиции вроде «Иешуа есть царь Иудейский, или он сегодня повиснет на кресте» могут делать истинными несколько факторов истинности: тот факт, что ты царь, тот факт, что тебя казнят, или даже оба вместе. Но как быть, например, с отрицательными высказываниями вроде: «Нет такого закона, который Иешуа бы нарушил»?

Иешуа: С этим у теоретиков факторов истинности действительно тяжело. Одни из них говорят, что фактором истинности отрицательных истин является второпорядковый «факт полноты», который указывает на все положительные первопорядковые факты в мире и говорит: это все подобные факты. [82] Другие постулируют отрицательные факты: некие реально существующие в мире наряду с вещами и их свойствами отсутствия вещей и свойств. [83]

Пилат: Какие-то сказки начинаются! Так в этом твоя теория, бродяга, в том, что истину объясняют сказочные невидимые факты отсутствия вещей?

§2.1.4. Остин: очищенная корреспондентская теория

Иешуа: В этой теории я и сам, игемон, пока сомневаюсь. У меня нет симпатии к отрицательным фактам, да и в целом теоретики факторов истинности берут на себя очень много онтологических обязательств. Некоторые говорят, что это ставит подход Валлийца и его последователей в трудное положение, и лучше держаться очищенной, как ее называет Петр Стросон из Лондиниума, версии этой теории. Ее разработал Иоанн Остин из Ланы Кастры. [84] Он верно считал, что для того, чтобы язык работал, нужен ряд условий, среди которых в вопросе истины нас особенно интересует одно. Это условие состоит в том, что должны выполняться два рода конвенций: а) дескриптивные (описательные) — они соотносят слова (предложения) с типами ситуаций, вещей, событий и т.д. в мире; б) демонстративные (указательные) — они соотносят слова (высказывания) с историческими ситуациями (то есть токенами ситуаций) и т.д. в мире. Одно может быть без другого: я знаю описательно конвенционально слово «кесарь», хотя в моей жизни не было исторической ситуации, когда я мог бы его правильно референциально употребить. С другой стороны, мы можем указать на твой, игемон, трон и назвать его именем «Соломон», но это не значит, что после этого слово «Соломон» будет обозначать всякий трон. Когда я говорю «в этом зале сейчас мало народа», то это мое высказывание, чтобы быть истинным, должно соотноситься с общепринятым, описательным способом говорить о залах, о количестве народа и т.п. Высказывание истинно, когда ситуация (токен), с которой оно соотносится указательной конвенцией, принадлежит к тому типу, с которым описательной конвенцией соотнесено предложение, которое используется для вынесения этого высказывания. Иными словами, высказывание истинно, если оно такое, какое у нас принято использовать в таких ситуациях.

Пилат: Получается, что мы решим, то и будет истиной. Это плохое учение!

Иешуа: Не получается: конвенция не создается одним человеком, но всеми людьми вместе. Ты, игемон, самый могущественный человек в Иудее, а и ты не можешь просто решить, чтобы чашу теперь называть копьем, — так чтобы все тут же стали ее так называть.

Пилат: Поберегись, бродяга, таких дерзких примеров! Решить, что чаша теперь копье, я, может, и не могу, но вот твою судьбу решать буду именно я. Тем более, что ты мошенничаешь: говорил, что это теория соответствия, но что же тут соответствует чему?

Иешуа: Но я совсем не обманывал: высказывания соответствуют ситуациям и типам ситуаций.

Пилат: Это-то я услышал. Но что это значит, где здесь соответствие? Вот у Рассела мне это было понятно: убеждение и факт отражают друг друга, они буквально друг на друга похожи по своей форме — почти совсем как у Аристотеля. А тут как понимать твое соответствие?

Иешуа: Это ты верно заметил, игемон, тут соответствие совсем другое. Когда мы говорим о соответствии высказываний типам ситуаций, соответствии, результатом которого будет истинность утверждения, — речь идет об абсолютно и всецело конвенциональном соответствии. [85] А конвенций относительно одного и того же типа ситуаций — например, того, что слева от тебя секретарь, — может быть много. Поэтому неправда, что каждому факту, или ситуации, можно поставить в соответствие только два высказывания о нем: утвердительное и отрицательное — как говорил Валлиец. Язык не должен никак «отражать» мир. А доказать это очень легко: простой язык может иметь лишь очень бедные выражения для описания чрезвычайно сложных фактов (подумай, например, о грязной брани), а сложный язык может иметь сложные выражения для простых фактов. Но это не делает второй более пригодным для выражения истины, чем первый. Эллинский философ скажет, что твой секретарь сидит слева от тебя в дюжине слов, а добрый человек Марк Крысобой, может, справится тремя, но это не будет значить, что грек сказал нечто более истинное, чем кентурион.

Пилат: Так что же в итоге ты думаешь об истине, бродяга? Думаю, ты путаешься в том, что прочел и, видимо, плохо понял из-за своей глупости!

§2.1.5. Элстон: реалистическая теория

Иешуа: Я, игемон, скромный человек. Я лишь защищаю реалистическую концепцию истины. Утверждение истинно, если и только если то, о чем оно говорит, действительно имеет место; содержание утверждения — все, что нужно для того, чтобы определить, что значит для него быть истинным. Концепция моя реалистическая, потому что истина зависит от мира и не является чем-то зависимым только лишь от свойств самого утверждения, от его познавательного статуса, обоснованности, гарантированности и тому подобного. [86]

Афраний: Позволит ли господин прокуратор мне держать слово?

Пилат: Конечно, дорогой друг. Видишь, что городит этот человек! Теперь вся надежда только на тебя! Открой же нам, есть ли преступление в его словах? Ведает ли он истину, которую взялся проповедовать?

Афраний: Я лишь хотел узнать у него, прокуратор, зачем говорить такие очевидные вещи? Ведь если бы некто стал повторять, что стол есть стол, а комната есть комната, доказывая слушателям, что он открыл великую истину, то его сочли бы не знающим, но безумцем. [87] А безумцы опасны.

Иешуа: Истина проста, тут я согласен. Вся проблема, добрый человек, в том, как правильно о том сказать. Ведь многие отрицают самое простое, и мужество философа в том, чтобы говорить о том, что есть. Я только сказал то, что знаю об истине наверняка. Выше я говорил об идеях Рассела и Остина, чьи взгляды мне близки, но согласен ли я с ними в деталях — этого я еще не знаю. Я знаю, что значит слово «истина», и дал выше его номинальное, как говорят ученые люди, определение. Его реальное определение, определение того, какие свойства делают истинным то, что истинно, — дело дальнейшего исследования.

§2.1.6. Особое мнение Хайдеггера

Афраний: Мне случалось читать донесения о рассуждениях варварского германского колдуна Хайдеггера из тех мест, что мы, римляне, зовем Маркианой Сильвой, а свевы — Черным лесом.

Каиафа (перебивает): И мне! Он ненавидит наш народ!

Афраний: Это все, что ты, уважаемый, понял из его работ?

Каиафа: Трижды я садился читать его, трижды засыпал на первой странице! У него только метафоры, нет аргументов! Сложно даже понять, выступает ли он за корреспондентскую теорию или против.

Афраний: Не стоит, Каиафа, кичиться неусидчивостью. Я, напротив, считаю, что проблема его трудов не в метафорах, их там как раз довольно мало, если сравнить с другими философами. [88] Проблема в том, что его труды, во всяком случае ранние, очень серьезные и систематические, и то, что он говорит в них об истине, сложно понять вне картины его философии в целом. Не думаю, что его рассуждения можно рассматривать как свидетельствующие «за» или «против» той теории истины, которую мы сейчас обсуждаем. Но он пытается показать, как эта теория коренится в том, что он называет фундаментальной онтологией. Я, впрочем, не уверен, что корреспондентская теория действительно требует такого укоренения, и не понимаю, почему он не рассматривает в свете своего подхода другие теории.

Анна: А в чем суть его подхода?

Афраний: Скажу так: он говорит, что анализирует некие данности. Но то данности не сознания, а бытия. [89] Применительно к истине получается вот что. Как связаны суждение и мир? Их связь — не сходство, тут Хайдеггер согласен с твоим, бродяга, Остином. Рассмотрим такой случай. Картина висит криво. Я считаю, что это так, и я прав. Как это возможно? Это возможно, если картина, как сущее, раскрылась мне в своем бытии, а я оказался раскрыт этому бытию. Истина — это раскрытость. «Раскрытость» — сложный термин. Сам Хайдеггер определяет его через «разомкнутость», которую он понимает как трансценденцию. [90] Когда нечто раскрывается, предмет показывает себя таким, какой он есть в своей трансцендентности [91] по отношению к нашему присутствию, которое и есть его разомкнутость. Далее колдун выделяет первичную и вторичную истинность. В первичном смысле истинно само присутствие как то, что раскрывает сущее, во вторичном смысле истинно то, что раскрываемо, т.е. это самое сущее или суждение о нем. Разомкнутость приходит в мир через присутствие. Это можно, думаю, понять через такую аналогию: римский народ есть власть в смысле источника власти (в первичном смысле), поэтому те, кого он этой властью наделяет будут властью (во вторичном смысле). Из этого следует, например, что до открытия законов движения материальных тел они не были ни истинными, ни ложными. Они не были даны, не были раскрыты — поэтому не были истинны. Но что не раскрыто, то не может быть и искажено — поэтому они не были и ложными. Ведь исток ложности — именно искажение раскрытости сущего. Метафизическая теория истины как корреспонденции, которую защищает арестант, возникает как субстантивация присутствия как лишь сущего. То есть в метафизическом смысле законы движения материальных тел могут быть истинны и без присутствия.

Каиафа: Я ничего не понял!

Иешуа: Я тут вижу, добрый человек, смешение генетического вопроса — об основаниях истины — с онтологическим — о сути истины...

Каиафа: А что ты имеешь в виду под сутью?

§2.1.7. Критика корреспондентской теории

Пилат: Оставим это. Ты сказал, бродяга, — я понял. Теперь вот что: я вижу пять основных проблем твоих корреспондентских теорий. Загибай пальцы: я буду последовательно высказывать их, а ты отвечай.

Иешуа: Да, игемон.

§2.1.7.1. «Этическое» возражение

Пилат: Неясно, как в рамках твоей теории устанавливается истинность этических, правовых и эстетических суждений. Кажется, что то, что делает их истинными, это что угодно, но только не факты. Или ты скажешь, что когда я говорю, что на свете не было, нет и не будет никогда более великой и прекрасной для людей власти, чем власть императора Тиберия, — то я вовсе не говорю чего-то истинного?

Иешуа: Я вижу три возможных ответа на твой вопрос. Во-первых, можно сказать, что такие суждения не истинны и не ложны, они просто выражают отношение говорящего к некоему факту. [92] Но не спеши, игемон, гневаться, в грубой форме это решение не проходит. Оно неудовлетворительно с точки зрения многих моральных философов, так как подменяет нормативный статус морали ее исключительно психологическим выражением. [93] Создает оно проблемы и для логики, так как приводит к проблеме смешанных высказываний. [94] Но этот подход можно улучшить в духе Теренция Хоргана, и это будет моим «во-вторых». Хорган считает, что на теоретическом уровне, морально невовлеченном, этические высказывания не истинны и не ложны, так как на уровне предельной онтологии, списка наиболее фундаментальных элементов сущего, морали нет. Но на практическом, морально вовлеченном уровне, этические суждения претендуют на объективность. Выносящий их не просто говорит «мне это нравится», а выражает готовность к приведению несубъективных доводов в пользу своей позиции. Утверждение морального дискурса — это языковой акт занятия позиции: через него человек выражает определенные моральные обязательства и занимает позицию по отношению к определенной проблеме. Хорган называет это «недискриптивистской когнитивной этикой». Это возможная позиция, более того, лучшая из возможных — если только ложен моральный реализм. [95] Но он-то и является моим «в-третьих». Моральные реалисты — это те, кто считает, что моральные суждения действительно направлены на то, чтобы сообщать факты, и они истинны, если они правильно представляют эти факты.

Каиафа: Но ведь говорят, что мудрец из племени скотов Давид Юм учил о великом принципе: из сущего нельзя вывести должное. [96] А если так, то из рассмотрения фактов, то есть сущего, никак нельзя вывести моральных истин, то есть должного.

Иешуа: Говорят-то говорят. Но это верно, только если морального нет уже в самих фактах, а это и есть то, что говорит моральный реалист.

Каиафа: Говорит-то говорит, да только что это значит? Что это за такие удивительные факты, в которых уже заранее содержится должное, и откуда мы можем их знать?

Иешуа: Строго говоря, добрый человек, моральный реалист отвечать на этот вопрос не обязан. Он скажет, что «трусость — худший из пороков» — это моральный факт, и в этом нет ничего противоречивого, а как мы его узнаём, и почему многим это неочевидно — это отдельная проблема; ведь и не все, страшно сказать, верят, что в Египте живут крылатые змеи. [97]

Каиафа: Это что же, игемон, он думает, что мне его наглые намеки невдомек? Он же мне сейчас просто сказал, что он перед нами не ответчик!

Иешуа: Нет, добрый человек, я о такой дерзости и не думал — я только иду, куда ведет меня истина. Истинно, что моральный реалист может в ответ на твой вопрос молчать. Но может, конечно, и ответить. Утверждать существование моральных фактов он может по крайней мере еще двумя способами. Во-первых, он может считать такими фактами моральные конвенции. То есть моральное суждение «рабство аморально» значит, что большинство разумных членов сообщества осуждает рабство. Так, я слышал, рассуждает Бодемус сын Валериана Василия из северных склавинов.

Каиафа (в ужасе косясь на Пилата): Рабство аморально? Но ведь это чушь!

Иешуа: Согласен, это сложно обосновать. Но второй подход обосновать еще сложнее: он гласит, что моральными фактами нужно признать элементы системы моральных обязанностей, которые следуют из чистого долга, предписанного нашим разумом самому себе. Таким образом, моральные факты суть факты самого разума.

§2.1.7.2. «Эпистемологические» возражения
§2.1.7.2.1. Простое «эпистемологическое» возражение

Пилат: Допустим. Второе возражение: как мы узнаём об истинности того, что истинно? Если истинность заключается в соответствии убеждения и факта, то узнать о ней невозможно, так как для этого бы пришлось сравнить пропозицию и факт, а их сравнить нельзя.

Иешуа: Тут, игемон, важно понять, что ты хочешь сказать. Если речь о том, что я понимаю, что истинно, что этот зал освещен, когда я понимаю, что он освещен, то возможность такого сравнения пропозиции и факта, кажется, не будет отрицать никто. [98]

Пилат: Не притворяйся глупее, чем ты есть. Я говорю о том, что нельзя приложить освещенный зал к словам «освещенный зал» и сравнить, совпадает ли одно с другим.

Иешуа: Ты хочешь сказать, игемон, что у нас нет возможности получить прямой доступ к фактам, с которыми мы должны сравнить убеждения. Но ничто не обязывает нас считать, что этот доступ должен быть прямым. Почему же он не может быть любым: прямым, индуктивно выведенным, дедуктивно полученным или еще каким-то? Все зависит от ситуации. Важно понимать, что корреспондентская теория не есть теория познания, она есть метафизика истины. И неважно, как именно мы схватываем истины; важно, чтó они такое и чтó делает их такими.

§2.1.7.2.2. Сложное «эпистемологическое» возражение

Пилат: Это недурной ответ. Но ты согласишься, что если метафизическая теория ведет к скептическим выводам, если из нее следует невозможность познания, то с точки зрения здравого смысла это плохая теория. Познание существует, и если из метафизики следует противоположное, то это плохая метафизика.

Иешуа: Как же не согласиться, игемон.

Пилат: И при этом ты говоришь, что убеждение истинно, если оно соответствует реальности. Допустим, так. Подойди-ка сюда, у меня тут чертежный песок. Давай для наглядности обозначим истинное убеждение литерой х, а факт, которому оно соответствует — F. Тогда получается, что, чтобы знать х, мы должны знать, что х соответствует F. Но «х соответствует F» — это новое суждение. Назовем его у. Чтобы оно было истинным, оно тоже должно соответствовать своему факту — тому факту, что х соответствует F; назовем этот новый факт F2. Значит, чтобы знать у, мы должны знать, что у соответствует F2, где «у соотвествует F2» — это опять новое суждение. И так далее. Получается, что теория соответствия ведет к регрессу в дурную бесконечность. [99] И это как раз делает познание невозможным.

Иешуа: Это сложное возражение, но мне оно не кажется верным. Кто возражает как ты, игемон, тот говорит, что, чтобы знать х, мы должны знать, что х — истинно. Но ведь это не так: знать — это просто иметь истинное обоснованное мнение; так говорит афинянин Платон, [100] лучше которого, пожалуй, никто о знании не сказал. Если я полагаю, что твой секретарь сидит слева от тебя, на том основании, что просто вижу это, и твой секретарь действительно сидит по твою левую руку, то этого уже достаточно, чтобы утверждать, что я это знаю. Нет нужды дополнительно требовать, как ты это делаешь, чтобы я вдобавок точно знал о том, что это мое убеждение истинно. Различие, которое упущено в твоем возражении, — это различие между знанием и знанием об этом знании; это различие тонкое, но очень важное. Теория соответствия не исключает возможность знания, она исключает возможность абсолютно достоверного знания — но где нам с тобой, игемон, взять теорию, которая не только пообещает такое знание, но и сдержит свое обещание?

Пилат: Что ж, это ловко. Но, я вижу, тебе еще есть что сказать?

Иешуа: Истинно так, игемон. Я, во-вторых, не уверен, что указанная тобой бесконечность действительно дурна. В рамках познания мы ведь действительно потенциально бесконечно можем уточнять наши знания, но прагматические основания велят нам остановиться там, где это разумно сделать в каждом конкретном случае. Это не значит, что корреспондентская теория с необходимостью ведет к прагматизму в отношении познания, но они могут дополнять друг друга.

Пилат: Но не оказывается ли корреспонденция тогда просто какой-то волшебной силой ума дотягиваться до реальности?

Иешуа: Не вижу тут волшебства. Мы не находимся вне мира, а значит, связаны с ним тысячью нитей, пусть иногда и незримых. Одной из них является корреспонденция убеждений и фактов. Тут все зависит от того, как именно понимать корреспонденцию. Мы с тобой, игемон, уже говорили, что одни мудрые люди понимают ее через причинность, другие — через изоморфизм, третьи — через конвенцию. Но если причинность — это волшебство, то вся физика волшебна. Если изоморфизм — это волшебство, то эллинские геометры — маги. Если конвенция — волшебство, то я уж и не знаю, что значит это слово!

§2.1.7.3. Факты?

Пилат: Поаккуратнее, бродяга! Если я даю тебе говорить, это еще не значит, что я буду терпеть твою дерзость. Помни об этом и скажи: если корреспонденция не кажется тебе загадкой, то что ты скажешь о фактах? Существуют ли факты? Доступны ли они нам?

Иешуа: Почему нет? Ты, игемон, упомянул здравый смысл, он вполне определен в этом отношении.

Пилат: Да, но что делает факт доступным? Мы получаем доступ к фактам только через язык. Факты потому, могу сказать я, есть лингвистические конструкты, такие же как и суждения. У них одна природа — языковая.

Иешуа: В твоем рассуждении, игемон, боюсь, нет следования между посылками и заключением. Из того, что мы имеем доступ к фактам только через описание фактов, не следует, что факты и их описания имеют одну природу. Со своей стороны, я могу привести конструктивный аргумент в пользу утверждения о различии их природы. Это различие попросту легко обнаружить: из стакана с водой, стоящего перед нами на столе, можно, например, пить, а из его описания — нельзя. Опыт взаимодействия с фактами и с их описанием различен.

Пилат: Однако и из того, что различие фактов и их описаний имеет место, не следует, что это различие касается их природы. И факты, и описания их, скажу я, могут иметь общую, идеальную или лингвистическую природу, но при этом различаться по форме. Что ты на это мне ответишь?

Иешуа: Допустим, ты прав, и следования тут нет. Но тогда мы просто попадаем в тупик: каждый из нас привел по конструктруктивному аргументу в пользу своей позиции, но в обоих этих аргументах, похоже, обнаруживается отсутствие следования. Каждый остается при своем; я — при теории корреспонденции.

§2.1.7.4. Теоретико-модельный аргумент

Пилат: Справедливо, хоть ты опять забываешься. Кто-то еще хочет возразить?

Анна: Я скажу, игемон, если ты позволишь.

Пилат: Говори.

Анна: Ребе Илларион Патнэм сказал: классификация теорий истины на корреспондентскую, когерентную, прагматическую и дефляционистскую — это катастрофа. [101] Все эти термины сбивают с толку. В частности, что такое корреспонденция? Видимо, такие как ты, бродяга, считают, что корреспонденция представляет собой некое уникальное отношение между фактическим положением дел и его описанием. Но допустим, что у нас есть два различных описания, две теории, и ב, относительно некоторого положения дел. Каждая из них полностью соответствует наблюдаемым фактам, каждая дает верные предсказания, но и ב постулируют существование разного типа объектов. Раз в таких случаях корреспондентская теория не помогает нам выбрать между этими теориями, то она никуда не годится! [102]

Иешуа: Но почему же, добрый человек? Как метафизическая теория она не должна помогать делать такой выбор. [103] Было бы, конечно, хорошо, если бы она делала еще и это. Но если не делает, то это не грех. Кроме того, я не уверен, почему тут нельзя сказать, либо что объекты, постулируемые , и объекты, постулируемые ב, — это одни и те же объекты, либо, если их различие является посылкой рассуждения, что это просто теории про разные объекты. Кроме того, этот довод не опасен для теорий в духе Остина, где отрицается требование единственности и уникальности корреспонденции, природой которой является простая конвенция.

(Анна хочет возразить, он явно недоволен простодушным невежеством Иешуа. Но Пилат прерывает его.)

Пилат: Я спрашивал, ты отвечал. Суд находит твои ответы неразумными. Но они находятся в рамках римского закона. Мы не предаем смерти за глупость. Однако суд еще не окончен. Я испытаю тебя своей теорией — посмотрим, как ты будешь отвечать. Если ты и тут не будешь говорить преступно, то тебя испытают мои помощники: Афраний, секретарь и Марк. Ты понял меня, бродяга?

Иешуа: Да, игемон.

§2.2. Когерентная теория
§2.2.1. Бланшард: сильная когерентность

Пилат: Начну. Брэнд Бланшард из Охиума учил меня когерентной теории истины. Он говорил, что мыслить — значит искать понимания. Мы желаем постичь то, что нам неизвестно, и поставить это в некоторое отношение к тому, что нам известно, достичь систематического, и в идеале — полного и исчерпывающего взгляда на мир. Таким образом, мы ищем такой системы знаний, которая в пределе была бы неотличима от самого мира. Но неотличимая от мира система не может быть ничем иным, как самим миром. Это идеалистической аргумент. Ты, вижу, хочешь спросить: что если удовлетворение этого желания даст нам систему убеждений, которая не будет соответствовать реальному миру? Отвечаю: если мы представляем мысли и вещи как находящиеся только во внешнем, поверхностном отношении, то, когда мы приведем мысли в систему, это, действительно, еще не даст нам знания вещей. Но такое понимание отношения мысли и предмета основано на неправильных аналогиях, с помощью которых мыслят знание. Его представляют как копию некоего оригинала. Мы же учим, что мыслить вещь — значит до определенной степени погрузить саму эту вещь в сознание. Если бы это было не так, то нельзя было бы надеяться преодолеть скептицизм, который растет из предположения о том, что суть знания — в копировании, ведь любая копия всегда чем-то отличается от оригинала, и точное знание было бы невозможно. Но прогресс великой римской науки показывает нам, что мнение скептиков безосновательно. Поэтому нужно признать, что слово «истина» значит «приближение мысли к реальности». Ее мера — расстояние, которое мысль преодолела на пути к идеальной системе. Мера истины опыта в целом — это мера систематичности, которой мысль достигла. Суждение истинно, если оно принадлежит к идеальной системе, если оно с ней когерентно, или согласованно. В полностью когерентном знании каждое суждение будет следовать из всей остальной системы и логически влечь эту систему за собой. Такова природа истины и природа реальности, потому что они в пределе суть одно. Конечно, мы можем ошибаться в том, что некое суждение действительно принадлежит идеальной системе. Но это лишь значит, что мы должны быть осторожны в исследовании и скромны в речах. [104]

Иешуа: Это интересно, игемон, но я пока вот чего не понял: все-таки много истин, или она одна? С одной стороны, кажется, одна, ведь истина состоит в полноте и тотальности. С другой, ты говорил и об истинности отдельных суждений. Тогда истин вроде бы получается много.

Пилат: Это спор о словах. Для практических нужд мы используем слово «истина» так, для теоретических и точных — иначе. Истина отдельного высказывания определяется через его соответствие с актуальным знанием. И истина этого знания как целого должна быть измерена приближением этого знания к абсолютной системе.

Иешуа: Ты говоришь о приближении знания к идеальной системе. Но если знание должно соответствовать какому-то внешнему по отношению к нему идеалу, то не вернулись ли мы обратно к корреспонденции?

Пилат: Хитро, но нет: наше знание не копирует систему, а становится ей; сближение актуального знания и идеальной системы — не внешнее, а внутреннее отношение.

§2.2.2. Критика когерентной теории. Аргумент двух систем

Иешуа: Главным аргументом против когерентной теории истины, я думаю, будет аргумент двух систем. Он звучит так: пропозиция может принадлежать к некой когерентной системе, не будучи истинной... [105]

Пилат: Вот так аргумент! Ты тут просто сказал: ты, Бланшард, не прав!

Иешуа: Позволь мне закончить, игемон. Вот почему: когерентных систем убеждений может быть несколько, а коль скоро природа истины есть когеренция, то наша теория не предоставляет возможность выбрать между двумя системами. Тогда, допустим, суждение S может истинным и ложным одновременно, если оно принадлежит одной системе и не принадлежит другой. Скажем, в нашем мире ложно, что афинянин Сократ был софистом, но это истинно в мире пьесы его соотечественника Аристофана «Облака».

Пилат: Но когерентная теория не утверждает, что любая когерентная система истинна. Она прямо утверждает, что такая система только одна: полная, идеальная система знания. Мир Аристофана не является таковой.

Иешуа: Но почему, игемон, не может быть нескольких полных систем?

Пилат: Их сосуществование, бродяга, было бы просто невозможно. Допустим существуют две разные системы, равным образом объемлющие все имеющиеся в мире факты. Если это разные системы, а факты в них одинаковые, то они должны отличаться чем-то помимо фактов. Единственное, чем они могут отличаться — это структурой: у первой системы будет структура А, у второй — В. Но того факта, что у второй системы структура В, нет в первой системе, а того факта, что у первой системы структура А, — нет во второй. Поэтому ни одна из них на самом деле не объемлет все факты. [106]

Иешуа: Это не лишено смысла. Но почему в первой системе нет факта относительно структуры второй системы, и наоборот? Ведь такой факт о другой системе может содержаться даже в неполных системах.

Пилат: Ты, несчастный, опять ничего не понял — а все потому, что все еще держишь в уме различие фактов и суждений. Давай я скажу это на твоем языке. В полной системе истинных суждений А есть суждение (1) «эта система имеет структуру А», а в полной системе истинных суждение B есть суждение (2) «эта система имеет структуру B». Суждение (1) не может принадлежать B, потому что там оно будет ложным, а система B есть система истинных суждений. То же верно и для суждения (2) и системы А. А это делает обе эти системы неполными.

Иешуа: Вот как! Это более хитрое возражение, чем твое первое. Однако меня волнует, что высказывание «эта система имеет структуру такую-то» двусмысленно, потому что содержит выражение «эта система», которое может значить две разные вещи. Оно может значить «та система, в которой сейчас употреблено это выражение». А может быть так, что слова «эта система» имеют в полной системе истин четко фиксированное значение и в рамках этой системы значат только, например, «система А».

Пилат: Я думаю, что мы должны выбрать второй вариант, так как термины в любом рассуждении должны быть строго определены и не должны менять своего значения в рамках одного рассуждения.

Иешуа: Хорошо, давай зафиксируем значение этого выражения. В системе А выражение «эта система» мы будем понимать как имя, обозначающее систему А. Тогда выражение «Система А» материально эквивалентно выражению «эта система»: всегда, когда сказано первое, на его место может быть поставлено второе без изменения смысла и значения высказывания. В системе В выражение «эта система» будет именем системы В. Чтобы впредь не путать, давай говорить об А «эта система-1», а о В — «эта система-2». Тогда в А содержится суждение (1*) «Эта система-1 имеет структуру А», а в В — (2*) «Эта система-2 имеет структуру В». Тогда ничего не мешает нам сказать, что (1*) принадлежит также и В, так как оно эквивалентно суждению «А имеет структуру А», а все эквивалентные суждения следуют друг из друга. То же верно и для (2*) и системы В. Эти истины содержатся в обеих системах. Тогда у нас будут две полные системы с разной структурой, и когерентная теория не сможет сделать между ним выбор. Согласен ли ты с этим?

Пилат: Проклятье. Ну, допустим, согласен.

Иешуа: Теперь рассмотрим предположение, согласно которому значение выражения «эта система» не является жестко фиксированным. Выражение «эта система» относится к той системе, в которой оно содержится: оно представляет собой функцию, которая сопоставляет высказывание системе, внутри которой оно находится. Но тогда не может быть и одной полной системы. Допустим, что существует система С, подсистема системы А. С — не полная система. Но неполная система — это тоже система. Допустим, в С содержится суждение (3) «эта система имеет структуру С». Принадлежит ли (3) системе А? Если оно принадлежит системе А, то оно ложно, так как система А имеет структуру А, а не С. А — система истинных суждений, ей не может принадлежать ложное суждение. Значит, (3) не может принадлежать системе А. Но если оно не принадлежит системе А, то оно тоже ложно, так как А, по предположению, выражает всю совокупность истины. Но, по допущению, С есть подмножество А, и все суждения С входят в А. Суждение (3) входит в С, значит, оно входит и в А и, по определению А, является истинным. Эта линия рассуждения тоже приводит нас к противоречию. Значит, даже одной полной системы не может существовать, если мы допускаем в нее такие контекстно-зависимые выражения.

Пилат: Лихо. Но что мешает Бланшарду сказать, что в случае C и А слово «система» используется в разных смыслах?

Иешуа: Думаю, что слова «система» в выражениях «система А» и «система C» имеют разные коннотации, но все же Бланшард, как я тебя понял, не запрещает нам говорить о неполных системах. Отвечая на довод про пьесу, ты сам говорил о таких системах. Для любой когерентной системы (хотя когерентист считает, что вполне когерентная система может быть только одна) верно, что она развивается экстенсивно, то есть в нее постоянно включаются суждения, и ни в один момент ее развития никакое суждение, однажды попавшее в систему, не может быть из нее исключено. Но если так, то это значит, что система нашего знания на этапах, предшествующих финальному, хотя еще и была неполной, но все-таки уже была системой.

Пилат: Хорошо. И все же я не убежден. Ты говоришь, что, по предположению, значение выражения «эта система» зависит от того, где оно использовано. А между тем у нас тут два разных контекста: А и С. Изменение контекста ведет к изменению значения, т.е. в рамках С в выражении (3) «эта система имеет структуру С» «эта система» значит «С», а в рамках А — «А». Это значит, что это два разных суждения, и парадокса не возникает. В первом случае указанное выражение просто не будет отсылать к А.

Иешуа: В отличие от случая двух систем, тут высказывание (3) автоматически принадлежит и к С, и к А, так как C — это подсистема А. Если мы в принципе допускаем подсистемы в полной системе, то невозможно отрицать, что (3) — это одно и то же суждение что в А, что в С. Оно, быть может, отличается на уровне смысла, что и делает его то истинным, то ложным. Но оно само, как факт, меняться не может.

Пилат: Мы можем тогда снова вернуться к предположению, что значения выражений типа «эта система» жестко зафиксированы. В таком случае (3) в С эквивалентно суждению «С имеет структуру С», а (3) в А, которое я буду называть (3*), эквивалентно суждению «А имеет структуру С». Тогда получается, что (3*) ложно и просто не принадлежит А. Это позволяет избежать разрушения системы, потому что мы опять имеем дело с двумя разными суждениями.

Иешуа: Но тогда мы возвращаемся к возражению от двух систем. Допустим, жесткая фиксация значения выражения «эта система» позволяет тебе блокировать мое рассуждение против существования единственной полной системы истин тем образом, каким ты сейчас это пытался сделать. Но тогда в случае (1*) и (2*) мы также должны понимать выражение «эта система» как жестко зафиксированное, и тогда, как я показал выше, а ты, игемон, согласился, у нас будет две полные системы, что недопустимо для твоей теории.

Пилат: Хорошо, пусть систем будет две. Но уверен ли ты, что нас должна волновать проблема выбора между ними? Кажется, что это эпистемический вопрос, а не онтологический.

Иешуа: В данном случае нет. Онтологически нет фактов, которые позволили бы предпочесть одну из них другой. Даже Бог не смог бы этого сделать, уж поверь мне. А это ведет к тому, что возможно суждение, которое одновременно будет и истинным, и ложным, потому что одной системе оно принадлежит, а другой — нет.

Пилат: Когда Анна критиковал тебя с помощью довода Патнэма, ты сказал, что нельзя метафизические тезисы ругать с помощью эпистемологических доводов. А теперь ты сам поступаешь именно так. Нехорошо, бродяга!

Иешуа: Так ведь у корреспондентской теории нет эпистемических обязательств, она ничего не говорит о познании. Твоя же теория говорит о знании много, вспомни свою собственную речь!

Пилат: Ну не то, чтобы это была моя теория, но я принимаю твои слова. Тогда пусть далее говорит Афраний, а ты отвечай.

Иешуа: Да, игемон. [107]

§2.3. Эпистемические теории

What we say is here already true in advance of
special verification...
James

Афраний: Я скажу как раз о тех теориях, которые связывают истину и знание. Люди моего ремесла придерживаются обычно одной из трех теорий истины. Все три теории объединяет следующий тезис: истина — это эпистемически нагруженное понятие. Поэтому теории такого типа иногда называются эпистемическими. Их сторонники считают, что истина неразрывно связана со знанием и обоснованием. Именно это отрицают сторонники корреспондентской теории, которую ты, разбойник, защищаешь: вы считаете, что нечто может быть истинным и без того, чтобы мы могли об этом узнать. Первая из наших теорий утверждает, что истина — это «согласие абстрактного положения с идеальным пределом, к которому бесконечное исследование будет стремиться привести научное убеждение». [108] Истина есть свойство пропозиций, которые эти убеждения выражают. Эту теорию мы называем теорией предела. Вторая теория исходит из отрицания подобной онтологии истины. Ее сторонники утверждают, что истина — это не свойство суждения, а событие, которое происходит с некой идеей. Это событие — процесс подтверждения, подкрепления, проверки и усвоения идеи. [109] Так как эта теория говорит об идеях, мы называем ее психологической теорией. Эти две теории также называются «прагматическими»: Карл Пирс и Вильгельм Джеймс, от которых нам известны эти теории, зовутся прагматистами. Третья теория, излагаемая часто большими конспираторами, состоит в том, что частью понятия истины является то, что обычно нашей целью является вынесение истинных суждений, [110] а высказывание истинно тогда и только тогда, когда не обнаруживается таких положений дел, которые исключаются содержанием этого высказывания, и ложно, когда хотя бы одно такое положение дел обнаружено. [111] Эту теорию называют верификационистской теорией истины. Ее иногда тоже называют прагматической, хотя ее автор, Михаил Даммит, учивший в Оксонии, сам себя прагматистом не считал. Расскажу о каждой из них.

§2.3.1. Пирс и Патнэм: Истина как согласие с пределом исследования

Афраний: Пирс приходит к своей теории истины, пытаясь прояснить понятие реальности. Кажется, что самое близкое к нам, то, с чем мы должны быть знакомы непосредственно, т.е. реальное, есть нечто, черты чего независимы от того, как их представляет кто бы то ни было. Однако реальное — не единственная причина возникновения у нас убеждений, поэтому встает вопрос о том, чем отличаются убеждения о реальности, т.е. истинные убеждения, от убеждений о фикции, т.е. от ложных убеждений. В строгом смысле идеи истины и ложности принадлежат только и исключительно научному методу установления мнения. Наука работает, исходя из надежды приблизиться к истине, и действительно шаг за шагом к ней приближается. А истина есть то мнение, которому суждено быть в конце концов признанным всеми, кто производит исследование. Поэтому истина — это такое мнение, которое никем не будет опровергнуто — возможно, только уточнено, — и потому никогда нас не подведет. Объект, отраженный в этом мнении, и есть реальность. Поэтому реальность независима от мысли отдельного человека, но не от мысли всего исследовательского сообщества в целом. Истинное убеждение одновременно соответствует реальности и принадлежит пределу исследования. [112] Достопочтимому Анне будет, быть может, интересно узнать, что понятие предела научного исследования было переосмысленно ребе Патнэмом в понятии идеальных условий проверки. [113] Он полагает, что суждение истинно, если и только если оно может быть подтверждено в идеальных эпистемических условиях, и ложно, если в таких условиях может быть подтверждено его отрицание. Истинность зависит не от конкретного обоснования, но от возможности обоснования. Отрицание необходимой связи между обоснованием и истиной ведет, согласно Патнэму, к метафизическому реализму, который защищает полную независимость реальности от сознания. Из такой независимости следует возможность тотального скептицизма: если реальность независима от ума, то все наше знание может оказаться ложным. [114] Иных способов связать ум и реальность, кроме как связать истину и обоснованность, Патнэм не признает.

§2.3.2. Джеймс и Рорти: Психологическая теория истины

Афраний: Теперь скажу о психологической теории, которую создал Вильгельм Джеймс. Его подход состоит из следующих элементов: (1) доктрина воли к вере; (2) доктрина старых истин; (3) тезис об удовлетворении; (4) тезис практических следствий. Доктрина воли к вере гласит, что часто, если не всегда, мы находимся в ситуации, когда рационально обнаруживаемых свидетельств недостаточно для того, чтобы принять или отвергнуть некое убеждение, и при этом мы не можем просто отложить выбор между этими альтернативами и должны принять то или иное решение, потому что нам надо действовать здесь и сейчас. Единственным основанием для выбора тут является наша вера, которая определяет нашу волю в пользу принятия или отвержения убеждения. Доктрина старых истин помогает понять, почему в том или ином случае воля может подсказать нам принять или отвергнуть некое убеждение. Она утверждает, что некоторое новое убеждение, для того чтобы быть принятым, должно быть согласовано с набором убеждений, которые уже приняты. В случае несоответствия мы должны либо изменить наши убеждения, либо отвергнуть новичка. В выборе между этими альтернативами нам помогают элементы (3) и (4). Тезис об удовлетворении гласит, что принятое в качестве истины убеждение должно помогать нам удовлетворять наши потребности, во всяком случае в долгосрочной перспективе. Тезис практических следствий гласит, что понимание какой-то мысли возможно только благодаря рассмотрению событий, причиной которых является предмет этой мысли. Иными словами, Джеймс связывает мысль и реальность, но не через корреспонденцию, а через проверку этой мысли. Таким образом, истинность некоего убеждения ставится в зависимость от нашего волевого решения, от когеренции с уже принятыми в качестве истинных убеждениями, от способности убеждения быть нам полезным и от проверки его надежности на опыте. Истина — это не застывшее свойство идеи, это то, что с идеей случается в процессе ее проверки и принятия. В этом смысле истина не статична и постоянно меняется, как и сама реальность. Большим достоинством этой теории является тот факт, что она объясняет не только то, что есть истина, но и то, почему мы к истине стремимся.

Иешуа: Связь мысли и реальности через событие проверки — это толково. Но разве можно таким образом проверить глубоко абстрактные идеи — например, положения математики или законы логики?

Афраний: В этом вопросе Джеймс согласен с Юмом: нужно различать истинность убеждений о внешней реальности и истинность убеждений об отношении идей. Математика и логика работают с последней. Мы сравниваем одни логические идеи с другими и упорядочиваем их в системы. Это упорядочивание не нуждается в непосредственной проверке: если мы правильно упорядочили идеи, то получили истину.

Иешуа: Не получается ли тут круга: разве правильность упорядочивания идей не зависит от истины, которая тут через них определяется?

Афраний: Не обязательно. Мы можем понимать правильность тут как соответствие правилам, принятым в этой конкретной математической или логической системе, — скажем, принципу непротиворечия или правилам деления. При этом выбор самих этих правил может быть мотивирован уже чисто прагматическими соображениями.

Иешуа: Но при этом математические истины он считает тоже относительным и допускающими пересмотр?

Афраний: Тут, признаюсь, не все ясно. Джеймс прямо говорит, что истинность убеждений, касающихся отношений между идеями, носит абсолютный и безусловный характер. [115] Однако как это стыкуется с его общей теорией истины, неясно. Другая трудность состоит в самой природе математического знания. С одной стороны, Джеймс утверждает, что истинность математических положений зависит от того, как мы упорядочили наши идеи. [116] С другой, буквально в том же месте он утверждает, что правила математического мира, следующие из самой структуры нашего мышления, носят по отношению к нашей воле принуждающий характер, такой же как и данные чувственного опыта. [117] Как согласовать эти утверждения в рамках теории Джеймса, я не знаю, однако мы могли бы вместо того, чтобы связывать математику с глубинами нашего мышления, утверждать конвенциональность ее природы.

Иешуа: Могли бы.

Афраний: Это было бы в духе Рикардуса Рорти из Нового Эборака, который радикализировал подход Джеймса. Поскольку у последнего истина оказывается неотличима от обоснования, то Рикардус предложил вообще отказаться от истины как от лишнего слова, оставив одно лишь обоснование. Лучшее, на что годится понятие «истина», — это использование его в качестве похвалы для того, что мы одобряем. Поэтому нам не нужна какая-то специальная философская теория истины. [118]

Иешуа: Но разве не говорим мы иногда: «это обосновано, но, возможно, не истинно»?

Афраний: На это Рорти бы ответил, что термин «истина» в таких выражениях используется только для того, чтобы показать, что все приводимые нами обоснования являются успешными доказательствами только для некоторой конкретной аудитории, и возможно, что для другой, более внимательной и образованной аудитории, то, что мы подаем как обоснование, не будет обосновывать то, в пользу чего мы его приводим.

§2.3.3. Даммит: истина и верификация

Афраний: Если Джеймс, Рорти и Патнэм исходили в конечном счете из идей Пирса, то третья эпистемическая теория истины, созданная оксониенцем Михаилом Даммитом, исходит из идей другого великого логика, Гудилуба Фреге из Иэны.

Каиафа: Опять ты, Афраний, тащишь в свой рассказ врага нашего народа! Впору уже и на тебя жаловаться игемону!

Афраний (не обращает на Каиафу внимания): Фреге считал истину и ложность двумя абстрактными объектами, которые являются значениями предложений. Даммит заметил, что Фреге не может обоснованно утверждать, что, определив Истину и Ложь как референцию предложений, он тем самым полностью объяснил эти понятия и способ их употребления. Ведь он не указал, с какой целью мы вообще используем понятие «истина». Фреге словно бы задал правила настольной игры, указал, какие положения фишек в ней считаются выигрышными, а какие — проигрышными, но не определил, что наша цель — выиграть. Целью введения истины и ложности как значений предложений или высказываний, говорит Даммит, является то различие, которое они позволяют провести между двумя классами положений дел. Каждое высказывание S, если оно не является двусмысленным или туманным, делит все возможные положения дел на два класса. Для положений дел первого класса характерно, что (1) высказывание S исключает возможность их обнаружить; (2) если некто утверждает S и при этом рассматривает одно из положений дел первого класса как то, которое возможно обнаружить, то мы сочтем, что он либо заблуждается сам относительно смысла утверждаемого им высказывания S, либо хочет ввести в заблуждение свою аудиторию. Для положений дел второго класса характерно, что высказывание S не исключает возможность их обнаружить. Если обнаруживаются положения дел первого класса, то S ложно. Если положения дел первого класса не обнаруживаются (то есть все положения дел, с которыми мы сталкиваемся в опыте, — второго класса), то S истинно.

Я вижу, что ты, бунтовщик, не поспеваешь за моей мыслью, поэтому, так и быть, помогу тебе примером. Возьмем высказывание «Иешуа есть царь Иудейский». Содержанием этого высказывания будет содержание его возможного утверждения. Содержанием же этого утверждения будет набор ожиданий, который мы с ним связываем или предписываем другим связывать. Содержание же ожиданий определяется набором положений дел первого класса, то есть положений дел, появление которых в нашем опыте удивило бы нас. [119] Так, людей, считающих, что ты есть царь Иудейский, весьма бы удивило то положение дел, что ты — бродячий философ. Эта ситуация исключается тем утверждением, что ты — царь. Если некто принимал бы и это утверждение, и то, что ты — бродячий философ без кола и двора, то любой разумный человек счел бы это примером заблуждения или необычного употребления слов, ведь обычно цари не бродят по улицам, смущая народ, а живут во дворцах.

Иешуа: И все же это не значит, что цари не могут быть бродячими философами и могут не жить во дворцах. Возможно, твои ожидания тут, добрый человек, — это просто набор расхожих мнений и предубеждений. Но оставим это, я понимаю, что ты хочешь сказать.

Пилат: Но нам ведь, бродяга, важно не столько понимаешь ли ты сказанное, сколько вынужден ли ты с этим согласиться. Есть ли тебе что ответить на слова Афрания, или ты признаешь, что в истине до того ни черта не разумел?

§2.3.4. Критика эпистемических теорий. Проблема Мура

Иешуа: Что ж, игемон, я отвечу. Я думаю, что несложно показать, что истина не сводится к обоснованию, ведь класс истинных суждений не совпадает с классом обоснованных суждений. Во-первых, ясно, что многие суждения, которые были приняты в качестве обоснованных в древние времена, сегодня уже не считаются таковыми. Так, египтяне верили, что земля плоская, а мы, евреи, открыли, что она шарообразная, и от нас это узнали греки, которые выдали потом слово Моше за свой платонизм. [120] Разве изменилась от того форма Земли? Ты скажешь, что мнение египтян просто не было обосновано, но они-то держались иной точки зрения на этот счет. А чтобы задать более сильное понятие «обоснования», чем культурно-ограниченное, нам нужно реалистическое понятие истины. Во-вторых, теория истины, как мне думается, должна объяснять тот факт, что случайно сделанное высказывание порой оказывается истинным. Представим, что я стою перед мешком с куриными яйцами, не зная, сколько их там. Я говорю: «их там пять». Потом, по подсчету, выясняется, что я сказал истину. Разве не странно считать, что это подсчет сделал мои слова истинными и что до того они истинными не были? В-третьих, допустим, что много лет назад мы с другом играли в коттаб. Мы играли долго и в свое удовольствие, так что счета не вели, а память у нас слаба. При игре было только двое нас. И вот суждение: «Иешуа попал в коттабий семь десятков раз». Ничто на свете, кажется, не сможет доказать или опровергнуть это суждение, при этом оно должно быть либо истинным, либо ложным (по закону исключенного третьего), ведь я либо попал ровно семь десятков раз, либо нет. [121] Вот три моих довода о том, что истина не сводится к обоснованности.

Пилат: Что касается твоего первого довода, то он кажется мне просто смешным: ты сам заметил, что мы можем просто признать мнение древних об обоснованности их ложных суждений ошибочным, а то, что сильное понятие обоснования должно покоиться на реалистическом понимании истины, ты не показал, а просто заявил. Второй же твой довод, по-моему, сводѝм к третьему: один говорит об истинности необоснованных суждений, а другой — об истинности суждений, которые не только не обоснованы сейчас, но будто бы и в принципе обоснованию не поддаются. Так что именно на последний мне хотелось бы услышать ответ. Что скажешь, Афраний?

Афраний: На эти слова есть два ответа: ответ Пирса и ответ Даммита. Пирс считал, что нет оснований полагать, что существует вопрос, который не могло бы решить достаточно упорное и длительное исследование. Нам действительно может быть безразлично, сколько раз ты попал в коттабий на прошлогодней игре. Но, уверяю тебя, случись нам с тобой поисследовать эту проблему несколько часов в моей специальной комнатке для бесед, как все необходимые доказательства «за» или «против» твоего суждения будут собраны. Таким образом, твоя теория лишь запрещает исследования вопросов определенного типа без всякого на то основания. Даммит же считал, что в таких случаях, как твой, мы можем просто отказаться от закона исключенного третьего.

Иешуа: Ответ Пирса может быть лишь злой шуткой, а не серьезной философией. Ответ Даммита интереснее, но я не уверен, что вполне понимаю его основания: для отказа от такого важного принципа, как исключение третьего, нам нужны очень серьезные причины, а ты, добрый человек, их не привел. [122]

Пилат: Ну нет, на такие подробности у меня нет терпения. Спасибо, друг Афраний, но слово переходит дальше.

§2.4. Теория тождества

Секретарь: Теперь я буду говорить. Я испытаю тебя теорией истины как тождества. Этой теории когда-то придерживался твой Валлиец Бертранд, а перенял он ее от своего друга Георгия Мура из кантабригийских грамматиков. [123] Говорят, что позже ее держался и тот самый Фреге, [124] но точно ли это так — этого я сказать не могу. В самом общем виде она выглядит так: для каждой пропозиции p верно, что p истинна, если и только если p — это реальность, или факт.

Иешуа: Признаюсь, добрый человек, что я удивлен. Неужели кто-то верит в такой абсурд?

§2.4.1. Мур, Хорнсби, Дэвид

Секретарь: Но это ничуть не абсурдно и даже вполне согласуется с самым здравым смыслом. Посуди сам: когда мы говорим о то, что, скажем, «игемон сидит на троне», то что именно мы имеем в виду? Судя по всему, мы пытаемся что-то сказать не о наших мыслях об игемоне, не об абстрактном выражении положения «игемон сидит на троне», не о лингвистической форме этой абстракции, но о самой реальности, самом факте, что игемон сидит на троне. Поэтому когда мы говорим, что истинно, что игемон сидит на троне, то имеем в виду, что сам факт «игемон сидит на троне» истинен. И именно потому, что я говорю о факте, об элементе реальности, одинаково доступном всем нам, ты, бродяга, вообще только и можешь меня понимать. Ведь если мыслить содержание высказывания «игемон сидит на троне» психологически, т.е. как мою мысль о том, что игемон сидит на троне, то совершенно неясно, как мы вообще понимаем друг друга, ведь содержание суждений тогда оказывается индивидуально: в моей голове оно одно, а в твоей, дурной, конечно, совсем другое. [125]

Иешуа: Как странно. Мне-то, добрый человек, казалось, что ровно эта последняя мысль стоит как раз за корреспонденской теорией истины, и она вполне с ее объяснением справляется. Но ты, видимо, не согласен?

Секретарь: Ничуть не согласен! Мы, сторонники теории тождества, считаем, что нет на свете учения хуже корреспондентской теории: полную ложность оно сочетает с мнимой очевидностью. От Гвиневер Хорнсби мне известно рассуждение, [126] которое не просто опровергает корреспондентскую теорию, а показывает, что ее единственной альтернативой является теория тождества. Допустим, истина — это корреспонденция между мыслью и миром. Тогда эта корреспонденция может быть либо полной, либо неполной. Она не может быть неполной: если корреспонденция неполная, это значит, что эта мысль и мир соответствуют друг другу только в какой-то степени; но истина не может иметь степеней: мысль не бывает чуть-чуть истинной или очень сильно истинной; истина должна быть, если мне будет позволено такое сравнение, как смерть, — все или ничего! Если же корреспонденция между мыслью и миром полная, то они просто совпадают — тогда корреспонденция неотличима от тождества.

Иешуа: Но почему же мы не можем сказать, что истина — это совпадение мысли и факта в одном конкретном отношении? Вот, например...

Секретарь (перебивает его): Но в каком именно? Мы должны будем выбрать какое-то конкретное отношение, и далее все повторяется: если вещи в нем совпадают полностью — то они тождественны, если частично — то это не истина!

Иешуа: Не серчай на меня, добрый человек, но это, по-моему, опять странный аргумент. Ты говоришь, что две вещи не могут полностью совпадать в каком-то отношении без того, чтобы быть одной и той же вещью. А я вот не вижу такой проблемы. Посмотри на подбой плаща игемона. Разве он не в точности того оттенка, что вчерашнее вино, от которого еще гудит голова Афрания? Плащ и вино полностью совпадают в отношении цвета, но это не сделало их одной вещью. Факт и мысль могут совпадать так же: скажем, как я и предлагал, в отношении формы, или еще в каком-то ином.

Секретарь (недовольно): Я совсем не это имел в виду, я имел в виду вот что. Допустим, мы утверждаем, что истина есть дело некоего отношения между высказываниями и миром — например, соответствия. Тогда, чтобы знать высказывание «истинно, что р», мы должны знать, что это отношение имеет место. Но, чтобы знать само высказывание «р», нам, кажется, вовсе не обязательно устанавливать наличие этого отношения: ты, бродяга, знаешь много истинных высказываний, но ведь ты не можешь сказать, что для того, чтобы их узнать, ты каждый раз усердно устанавливал соответствие. Вот и получается в твоей теории, что у «р» и «истинно, что р» разные эпистемические условия: можно знать одно, не зная другого, и наоборот. Но ведь это совсем не так: знать, что р — значит уже знать, что р истинно. [127]

Иешуа: Я не уверен, что вполне тебя понимаю. Но мне кажется, это твое возражение очень похоже на третье возражение игемона. [128] И я думаю, что ответ на него тоже похож на тот ответ. Да, я утверждаю, что знание отличается от знания о знании: знать «р» — не то же самое, что знать «истинно, что р». Можно обладать первым знанием, не обладая вторым. При этом я скажу, что каждый раз, когда я узнавал истину о том, что р, она была истинной именно в силу отношения соответствия, но вот мне самому об этом было знать необязательно. И последнее: допустим, что этот аргумент верен. Но почему он тогда не распространяется на тождество, ведь это тоже отношение? Его точно так же можно установить и при этом все еще сомневаться, истинно ли некоторое высказывание. [129]

Секретарь: Хорошо. Допустим, ты прав. Тогда предложу тебе другой аргумент. [130] Лучший путь к теории тождества — через наблюдение за тем, как работают изъяснительные придаточные предложения (вида «S верит, думает, считает и т.п., что р»), когда мы рассуждаем об истине и о фактах. Рассмотрим предложение «истинно, что р». Может показаться, что его логическая форма такова: «истинно что: р», где р — это предложение, а «истинно что» — это применяемый к этому предложению логический оператор вроде операторов негации, конъюнкции или дизъюнкции. Но теоретик тождества скажет, что такое понимание логической структуры этого выражения неверно. На самом деле выражение «истинно, что р» имеет логическую структуру «что р — истинно», где «что р» представляет собой имя пропозиции, выражаемой предложением «р», а «истинно» — это предикат, приписывающийся «что р». Такие выражения, как «что р», утверждаю я, всегда являются именно именами пропозиций — не только в таких выражениях, как «истинно, что р», но и, например, в таких, как «тот факт, что р». Есть у меня и аргумент в пользу этого: если это не так, то нельзя корректно формализовать следование типа: «игемон верит, что р»; «истинно, что р»; следовательно, «игемон верит в нечто истинное»; между тем интуитивно ясно, что следование тут есть. Если все сказанное верно, то становится возможно следующее рассуждение. Примем, что (а) «Истинно, что р, если и только если факт, что р». Эту посылку ты, думаю, отрицать не будешь: это просто здравый смысл, да и корреспондентисты говорят то же самое. Поскольку мы вскрыли истинную форму суждений вроде «Истинно, что р», то мы можем перейти от (а) к (b): «что р — истинно, если и только если что р — факт». А поскольку мы согласились, что «что р» — это имя пропозиции, мы можем, квантифицируя по именным местам, просто обобщить (b) до основного тезиса теории тождества (ТТ): «Для всякого х верно, что x — истинная пропозиция, если и только если х — факт».

§2.4.2. Критика теории тождества. Проблема лжи

Иешуа: Остроумно. Но ведь ты забыл указать на еще одно наше допущение при переходе от (b) к (ТТ): что выражения типа «что р» имеют устойчивую референцию — то есть всегда реферируют именно к пропозициям. Но я как раз с этим не согласен: я считаю, что в одних контекстах выражения вроде «что р» отсылают к пропозициям, в других — к фактам внешнего мира. Боюсь, что тебе будет непросто привести доводы в пользу устойчивости референции таких выражений.

Секретарь: Но ведь и ты не приводишь доводов в пользу своего сомнения в этой устойчивости!

Иешуа: Тут ты прав. Но у меня есть другой аргумент против твоей теории. Вот что главное: кажется, что она не может объяснить существование лжи. Тождество мыслится как необходимое отношение, а высказывания могут менять свою истинностную оценку.

Секретарь: Некоторые теоретики тождества считают, что отношение тождества тут контингентно.

Иешуа: Тогда такая теория тождества говорит, что пропозиция существует вне зависимости от того, истинна она или ложна. Но факт, что р, не существовал бы, если бы пропозиция была ложной. Значит, факт, что р, не может быть тождественен пропозиции, что р.

Секретарь: Так можно доказать что угодно. Скажем, что мужья не тождественны женатым мужчинам. Допустим, Марк — муж Цецилии. Марк существовал бы, даже если бы не был мужем Цецилии. При этом возможно, чтобы у Цецилии не было мужа. Из этого не следует же, что Марк не тождественен мужу Цецилии, верно? Из этого следует, что тот, кто в действительности является мужем Цецилии, мог бы и не быть мужем Цецилии. Иными словами, если бы пропозиция р была ложна, то ее нельзя было бы правильно описать как «факт, что р» (так можно описать только истинные пропозиции), но сама она от этого не перестала бы существовать.

Иешуа: А как тогда описать ложные пропозиции?

Секретарь: Как не-факты. Это такие сущности, которые во всем похожи на факты, только не являются ими: например, «что стакан сделан из воды». [131]

Иешуа: Это интересно. Уверен, что можно придумать экстравагантную метафизику, которая бы обосновывала и это. Но что все-таки такое пропозиция, которая не факт? Мне казалось, что достижение теоретиков тождества состояло в том, что вы избавились от удвоения: вы получили возможность сказать, что когда я говорю об игемоне на троне, то говорю о факте, а не о какой-то таинственной сущности (мысли или чем-то таком), которая еще как-то особо связана с миром. А теперь оказывается, что одно удвоение вы просто поменяли на другое: вместо дуализма мысли и мира у вас вышел дуализм мира и не-мира. Не хочу тебя рассердить, добрый человек, но старый дуализм мне нравится больше.

Пилат: А ну-ка постой, бродяга. Ты недоволен секретаревой теорией лжи, и, скажу, не без основания. Но какое нам дело до того, худа ли его теория лжи: мы-то хотели узнать, хороша ли его теория истины!

Иешуа: Но, игемон, не может быть хороша та теория истины, которая не является вместе с тем хорошей теорией лжи. Бертранд Валлиец сказал об этом, как раз отказываясь от теории тождества. [132] Да и ваш Аристотель говорил, что для противоположностей — одно знание: что объясняет движение, должно объяснять и покой, и хорошая теория света должна быть одновременно хорошей теорией тьмы.

§2.5. Дефляционизм

The view should be that truth is nothing.
Michael Devitt

Воланд (кричит с балкона): Браво, браво! Вот и я всегда говорю, что самое тонкое место в теориях истины — это теория лжи!

Пилат: Я и забыл о тебе, дорогой Луций! Заходи скорее. Не хочешь ли присоединиться к нашей беседе?

Воланд: Благодарю. Охотно поучаствую в вашем поиске истины!

Пилат: Отвечай же, бродяга Назорей, ибо тебя испытывать будет сам сенатор Луций из благородного дома Плутонов.

Воланд: Мне, друзья, более всего по душе учение, которое люди называют дефляционизмом. Позвольте поведать вам о нем. Когда мы говорим «истинно, что император Тиберий есть владыка этого мира», мы имеем в виду лишь то, что император Тиберий есть владыка этого мира. Тем самым не предполагается, что это высказывание обладает некоторым особым существенным для него свойством «быть истинным» или что оно стоит в некой мистической связи с реальностью. Предполагается лишь то, что описываемое в нем положение дел имеет место. В этом выражается центральный для всякого дефляционистского подхода тезис о «прозрачности истины». Иначе его можно выразить утверждением о том, что предикат «истинно» по существу ничего не добавляет к высказыванию, будучи ему приписан. Вот сейчас мы судим Иешуа. Если мы установим, что он виновен, то нам не придется дополнительно доказывать и иное высказывание «истинно, что он виновен». Тот факт, что значения у высказывания «А» и «истинно, что А», как видно из примера с императором, совпадают, и тот факт, что доказываются они одинаково, что видно из примера с доказательством вины, дают нам право утверждать, что высказывания «А» и «истинно, что А» просто эквивалентны. Такое полезное наблюдение за поведением слова «истина» позволяет нам работать с высказываниями так, как это делают в современной логике: заменять истинное высказывание на его истинностное значение и наоборот. Насколько я понимаю, впервые эта идея была высказана беспокойным стариком Фреге, хотя у него она и не легла в основание специальной теории.

Иешуа: То есть у твоих дефляционистов получается, что истина — это просто пустое, неинтересное понятие? Может, они предлагают совсем его упразднить?

§2.5.1. Рамсей: теория избыточности

Воланд: Не совсем. Прямо так думал, кажется, только Оттон Нейрат из Виндобоны. Кантабригиец Франциск Рамсей, с которого вся история дефляционизма по большому счету началась, впрочем, тоже склонялся к чему-то подобному. Он считал, что у убеждений есть т.н. пропозициональная референция, т.е. что каждое убеждение есть убеждение о чем-то, и это что-то представляет собой содержание этого убеждения. Кажется очевидным, что если человек убежден, что А есть В, и А действительно есть В, то его убеждение истинно. Проблема в том, что не все убеждения имеют форму «А есть В», поэтому эту очевидность проще всего обобщить так: «убеждение истинно, если это — убеждение, что p, и при этом p». Ложь тогда мы определим так: «убеждение ложно, если это — убеждение, что p, и при этом не-p». Ты мог бы сказать, что мы тут используем «не-», отрицание, для определения лжи, и это круг в определении. На самом деле нет: у слова «ложь», скажу по секрету, есть два значения — отрицание и ошибка. Здесь мы используем первый смысл для определения второго. Говорят, что Рамсей считал, что добавление слов «истинно» и «ложно» избыточно по отношению к высказыванию, к которому они применяются: их всегда можно просто опустить, и мы ничего от этого не потеряем. Такая теория зовется теорией избыточности.

Иешуа: Это явно не самая удачная формулировка того, что ты хочешь защитить. Если я тебя правильно понял, то главное в этой теории то, что «истинно» и «ложно» всегда можно убрать без потери смысла. Но рассмотри такое выражение: «Все, что сказал Отец наш Даниилу, было истинно». Тут явно нельзя взять и опустить слово «истинно»: если мы так поступим, то останемся просто с неправильным, неграмматичным выражением: «Все, что сказал Отец наш Даниилу, было».

Воланд: Согласен, большинство дефляционистов это признает. Более того, знал об этой проблеме и сам Рамсей, и его теория истины на самом деле сложнее теории избыточности, которую ему так часто приписывают. [133] Более того, неясно, является ли вообще теория Рамсея дефляционизмом, а не радикальной версией корреспонденской теории: ведь он считал, что хотя мы и не можем описать корреспонденцию в общем виде, не исключено, что в каждом конкретном случае истинность высказывания обеспечивается именно его соответствием реальности (за исключением, возможно, тех случаев, когда высказывание является сложным: например, дизъюнкциям вряд ли соответствуют особые дизъюнктивные факты). А это значит, что, хотя сам термин «корреспонденция» Рамсей считал подозрительным, он признавал, что высказывания имеют корреспондентские по своей сути условия истинности. [134]

Иешуа: Захватывающе. Надеюсь, к тому же здравому выводу о корреспондентской природы истины в итоге приходят и все остальные твои братья-дефляционисты?

§2.5.2. Стросон: перформативная теория истины

Воланд: Отнюдь! Например, Петр Стросон считал, что теорию истины нужно выстраивать просто из наблюдения за поведением слова «истина» в языке. [135] Именно это должно интересовать нас как теоретиков истины, а не условия, которые должны соблюдаться, чтобы наше использование слова «истина» было правильным. Ведь в языке слово «истина» употребляется и тогда, когда эти условия не соблюдаются. При этом Стросон не согласен с тем, что фраза «это истинно» является логически избыточной. Конечно, в языке есть множество способов утверждать, что А есть В, не произнося самой фразы «А есть В». Например, нас спрашивают «В ли А?», и мы отвечаем: «да». Вместо «да» мы могли бы сказать «это так», «это правда» или «это истина». В этих случаях мы используем слово «истина» и его производные как инструмент для сокращения выражений, потому что оно значит ровно то, с чем мы согласились. Мы также можем использовать слово «истина» и тогда, когда ближайший контекст разговора не подсказывает слушателю, что именно оно значит. Например, «то, что игемон сказал вчера, истинно». В этом высказывании нет информации о том, что вчера сказал игемон: «истина» здесь играет роль указания на некоторое и так известное слушателю обстоятельство. Такой анализ приводит Стросона к выводу: нам лишь кажется, что мы приписываем высказываниям статус «истина», тогда как на самом деле, используя это слово, мы делаем это неутвердительным перформативным способом. Примером похожего языкового явления будет приказ. Когда я приказываю тебе встать, я ничего не описываю, не отсылаю к некоторой реальности. Когда говорю, что «истинно, что ты стоишь», то я тоже ничего не описываю, я выражаю согласие и поддержку суждению «ты стоишь».

Иешуа: Мне кажется странным сам замысел такой теории. Какая разница, как мы употребляем слово «истина», если мы употребляем его порой неверно? Разве мы занимаемся лингвистикой, а не философией? Допустим, Стросон прав, и мы действительно употребляем слово «истина» так, как он описывает. В его теории нет ничего, что делало бы ее несовместимой с корреспондентским, когерентным или с эпистемическим подходом, так как те пытаются исследовать другой вопрос, а именно: чтó есть истина? Но даже если мы остаемся в рамках наблюдения за языком, мне кажется, очевидно, что можно обнаружить и другие способы использования слова «истина», помимо указанных Стросоном. Например, в логических выводах: «Если A, то В. Истинно, что A. Следовательно, В». Кроме того, мне приходит в голову еще такое соображение. По Стросону, сказать, что высказывание истинно — значит в конечном итоге выразить этому высказыванию поддержку. Но, кажется, не будет противоречивым такое суждение: «Хотя это и истинно, но я это не поддерживаю».

Воланд: Согласен, тут ты прав. Но я только начал.

§2.5.3. Куайн и Филд: истина как раскавычивающий инструмент

Воланд: Настоящую силу дефляционистские теории обретают в области логики. Это их истинное владение. Тут один из самых влиятельных подходов предложил Виллард Куайн, учивший в Кантабригии Массачусэсткой. Он обращается к схеме великого Эльфрэда Тарского из Варсовии: предложение «снег бел» истинно тогда и только тогда, когда снег бел. Как легко заметить на примере этой фразы, истина — это раскавычивание: приписать истину предложению «снег бел» — ровно то же самое, что приписать снегу белизну. Приписывание имени предложения, т.е. тому, что в кавычках, свойства «быть истинным», просто снимает кавычки. Поэтому в случаях, когда нам прямо дано предложение, которому мы хотим приписать истинность, предикат истины является языковым излишеством. В других случаях, когда не все предложения, о которых идет речь, даны нам непосредственно, он полезен. Прежде всего, мы не можем обойтись без него в обобщениях типа «Все, что говорит римский император, истинно» — просто потому, что чтобы сказать то же самое без использования подобного предиката, нам пришлось бы перечислить все, что император говорил, говорит и скажет, — а этого сделать нельзя. Таким образом, истина — это инструмент для составления такого рода бесконечных конъюнкций (и дизъюнкций), нужный потому, что у нас в языке нет других средств для их составления. Кроме того, как поясняет это Гартри Филд, это важно не только в естественном языке, но и при анализе теорий, положения которых выражены на языке логики предикатов первого порядка. При таком способе записи мы чаще всего записываем теории как набор аксиоматических схем (например, такой набор может включать схему математической индукции) и небольшого набора дополнительных аксиом. Некто принимает так записанную теорию, если соглашается с каждой из этих дополнительных аксиом и декларирует готовность принять каждый случай подстановки в аксиоматическую схему. Но некто также может и отвергать теорию, утверждая, что у нее есть неприемлемые следствия. При этом он может не знать, какая именно аксиома к ним привела. Тогда он будет выражать свой отказ в словах «Не каждая из аксиом этой теории истинна». [136]

Такая теория истины не дает строгого определения истины, ведь с ее помощью нельзя элиминировать термин «истина» во всех контекстах. Но, считает Куайн, она объясняет, что значит для предложения «быть истинным», и делает это в столь же простых, как и само предложение, словах.

Иешуа: Неужели нет другого способа объяснить работу бесконечных дизъюнкций и конъюнкций?

Воланд: Мудрые люди говорят, что это можно было бы попытаться сделать через подстановочную квантификацию. Но я не хотел бы утомлять супругу игемона объяснением того, что это такое. Для нас важно только то, что сама подстановочная квантификация зависит в своем определении от термина «истина». Поэтому без него в итоге все равно не обойтись. [137]

Иешуа: Кажется, этот дефляционизм тоньше предыдущих. Но все же я вижу в нем две специфические проблемы. Во-первых, если Куайн определяет истину через раскавычивание, то случаи раскавычивания как истины необходимо как-то отличить от других случаев применения этой процедуры. Ведь раскавычивание не является специфическим приемом, который описывает использование предиката истины, и только его. Мы можем сказать «Слово “вино” обозначает вино» — раскавычивание есть, но об истине речи не идет. Во-вторых, я не совсем понимаю, как Куайн решает проблему с адаптацией схемы Тарского к естественному языку. Для Тарского важно, чтобы его теория работала на конечном словаре, ведь только такой словарь может быть хорошо структурирован и формально определен. А в случае естественных языков мы явно имеем дело с бесконечным и плохо структурированным словарем.

§2.5.4. Гровер: Просентенциональная теория

Воланд: Не стану спорить. Может быть, тебе больше по душе придутся идеи Дорофеи Гровер. [138] Концепция Гровер, грубо говоря, состоит в том, что выражения типа «это истина» или «истинно, что» употребляются как просентенции, [139] ставящиеся вместо предложений, к которым они отсылают (как местоимения [140] ставятся вместо существительных, отсылая к ним). Например, некто сказал: «Сегодня вечером на Лысой горе будет интересная история!». И ему ответили: «Это истинно». Этот ответ служит просто способом повторить только что употребленное в разговоре предложение «Сегодня вечером на Лысой горе будет интересная история» без того, чтобы снова его произносить — точно так же, как местоимение «ты» служит способом сказать «Иешуа» без того, чтобы опять и опять звать тебя по имени. Важно, что в таком случае «это истинно» используется для того, чтобы сказать нечто о том, что случится вечером на Лысой горе, а не о свойстве предложения, которым выражена мысль об этом факте. В теории Куайна истина рассматривается как металингвистический инструмент, а в теории Гровер — как факт обыденного языка, как анафорический инструмент стабилизации нашего общения: она участвует в установлении понимания того, что мы с нашим собеседником говорим об одних и тех же вещах. Гровер спокойно относится к утверждению, что «истина» — это логический предикат, хотя оно и не является частью ее теории. Но она, как и любой другой дефляционист, резко отрицает, что истина — это субстанциальный, содержательный предикат.

Иешуа: Чем-то похоже на подход Стросона, только с уступками логикам. Но вот что меня волнует на протяжении нашего разговора о дефляционизме: не упускают ли его приверженцы в истине как раз самого главного? Разве истина не является ценностью для человека и целью познания? Разве, спрашивая об истине, мы хотим знать только логико-философские подробности употребления некоего термина?

Воланд: Прекрасный, прекрасный вопрос! Но Гровер есть что ответить. Нет большой беды согласиться, что истина обладает философской важностью! Мы с Гровер легко согласимся, например, с такими суждениями: «наше выживание зависит от нашего обладания знанием того, что истинно»; «цель исследования состоит в определении того, что истинно». Важно только правильно понимать выражение «то, что истинно». А оно отсылает к набору предложений, которые, в свою очередь, отсылают ко внеязыковой реальности. Таким образом, «то, что истинно» в тех утверждениях, которые я только что упомянул, подразумевает не более чем обобщение типа «Наше выживание зависит от обладания знанием того, что р, для многих р». Вот и все!

Иешуа: Что ж, это ответ. Но скажи, а как Гровер предлагает понимать выражения типа «это ложь»?

Воланд: Это выражение, очевидно, должно пониматься по аналогии с «это истина». Когда некто говорит «Иешуа есть царь иудейский», то ему отвечают: «Это ложь». То есть это все равно, что повторить «Иешуа не царь иудейский».

Иешуа: Ты говоришь «повторить», но в том-то и дело, что повторять нечего: никто ведь не говорил «Иешуа не царь иудейский».

Воланд: Понимаю! Я думаю, что мы должны тогда сказать, что оно все-таки отсылает к «Иешуа есть царь иудейский», только особым, отрицательным образом.

Иешуа: Но если «это ложь» есть просто просентенциальное выражение, то как же оно может отсылать к предложению, противоречащему тому предложению, которое оно в действительности пытается выразить? Если ему это как-то удается, то, чтобы это сделать, оно должно представлять собой нечто большее, чем простой просентенциальный инструмент!

Воланд: Охотно допускаю! Ибо разве в этом есть что-то страшное?

Иешуа: Если мы не принимаем всерьез требование Валлийца давать истине и ложности параллельные объяснения, то, наверное, нет. Но мне казалось, что если кто-то и должен беспокоиться о достойном осмыслении лжи, то именно ты.

§2.5.5. Хорвич: минимализм

Воланд: Что ж, если дефляционизм Гровер тебе не по душе, то позволь я расскажу тебе еще об одном, самом недавнем подобном учении: о минимализме Павла Хорвича, который и сегодня еще учит ему в Новом Эбораке. Много среди дефляционистов умеренных и взвешенных умов, но этот человек превзошел скромностью всех! У его теории нет многих амбиций других учений об истине. Он не стремится дать истине эксплицитного определения, не ставит себе целью дать необходимые и достаточные условия применимости истинностного предиката, не пытается дать редуктивную теорию свойства истинности и не намеревается предложить некий набор фундаментальных теоретических постулатов, на основании которых можно объяснить все прочие черты истины. Все, на что он претендует: объяснить значение слова «истинно». Объяснение тоже весьма скромное: значение предиката «истинно» полностью зафиксировано схемой эквиваленции: «Пропозиция, что p, истинна, если и только если p», или, что то же самое, «<p> истинно ↔ p». Если мы пониманием и принимаем эту схему, то мы понимаем, что значит «истинно», и в совершенстве умеем использовать этот предикат. Вот и все, больше в истине ничего и нет. [141]

Иешуа: Понимаю. Это опять, как и у Куайна, схема Тарского, только теперь она прочно поставлена в самый центр теории и слегка изменена — пропозиции вместо предложений. Но неужели ей вся теория и ограничивается?

Воланд: Ну не совсем. Дело не в ней самой, а в нашей готовности утверждать любые конкретные случаи этой схемы: именно эта готовность и лежит в основании значения предиката «истинно».

Иешуа: Но откуда берется сама эта готовность?

Воланд: Совершенно ниоткуда! Она ни от чего не производна и эпистемически фундаментальна. Должно же объяснение где-то заканчиваться!

Иешуа: Что ж, допустим. Но вот что меня беспокоит: если значение истины задается всеми случаями схемы «<p> истинно ↔ p», то, выходит, мы объясняем истину через бесконечный набор пропозиций?

Воланд: Именно так! Бесконечное множество конкретных случаев этой схемы будет набором аксиом минималистской теории истины. А страшного в этом ничего нет — для таких бесконечных естественных языков, как наши, такой их элемент, как предикат «истинно», конечным образом объяснить просто нельзя.

Иешуа: То есть эта теория на базовом уровне представляет собой просто перечисление всех случаев схемы «<p> истинно ↔ p»? Воистину скромное учение.

Воланд: Как я и обещал!

§2.5.6. Критика дефляционизма

Иешуа: Я выслушал тебя, и вот что я думаю. Дефляционизм — простая теория, которая хочет разрешить все проблемы метафизики истины одним махом. Такие подходы обычно сталкиваются со множеством технических возражений, на которые можно давать такие же технические ответы. [142] Но на них у нас не хватит ни времени, ни песка. Поэтому я сосредоточусь на менее технических, но более философских возражениях.

§2.5.6.1. Остин: аргумент мягкого факта

Иешуа: Первый аргумент я возьму у Остина. Он утверждает, что любое высказывание может в принципе отсылать к любой части мира (это зависит от конвенции), но не может отсылать к самому себе, иначе мы столкнемся с угрозой парадокса Лжеца. Тогда произвольное высказывание «S» может отсылать к миру или любой его части, за исключением самого себя, и высказывание «S — истинно» тоже может отсылать к любой части мира, за исключением самого себя. Легко заметить, что второе высказывание может быть корректно описано как отсылающее к первому, но при этом оно все еще не может отсылать к самому себе. Так как эти два высказывания отсылают к разным фактам, то это разные высказывания, и объявлять их эквивалентными, чего так хотелось бы дефляционизму, было бы поспешно. При этом высказывание «S — истинно» описывает факт, отличный от других стандартных фактов: факт, зависящий от нашей мысли (он полностью конвенционален), [143] а не «твердый факт», с которым мы ничего не могли бы сделать. Это — «мягкий» факт, подобный значению слов. Но все же это факт.

Воланд: Не впечатлен! Этот довод убедителен лишь в том случае, если нельзя победить Лжеца как-то иначе — а предложений на этот счет, право, масса. Еще мысли?

§2.5.6.2. Даммит: аргумент истинностных значений

Иешуа: Второй аргумент я возьму у Даммита. Он исходит из двух посылок, которые, кажется, должен принимать дефляционист. (1) «р» и «истинно, что р» — это одно и то же. (2) Предложения с пустыми именами не являются ни истинными, ни ложными, но являются бессмысленными. Из второй посылки следует, что выражения типа «дыр бул щыл» имеют истинностную оценку «бессмысленно», тогда как выражение «истинно, что дыр бул щыл» имеют оценку «ложь». Выражения с разными истинностными оценками не могут быть тождественными, значит, тезис эквивалентности, на котором стоит всякий дефляционизм, неверен.

Воланд: А вот это мне нравится, это мне очень нравится! Но вот только нечто вроде схемы эквиваленции ведь склонны принимать почти все теории истины — кроме, может быть, как раз верификационизма самого Даммита. И тогда мы с вами, корреспондентистами, в одной бочке: всем надо искать на этот прекрасный довод ответ! И ведь ищут! Вот и Хорвич иногда предлагает считать, что наше понимание истины — в готовности утверждать не все случаи схемы эквиваленции, а все ее несомненные случаи. [144]

Иешуа: Но он не делает себе услуги такой неясной добавкой. А что касается угрозы довода Даммита для других теорий, то они-то могут в крайнем случае от схемы эквиваленции отказаться, а вот от дефляционизма без нее ничего не останется.

Воланд: О, мне хотелось бы посмотреть на твои теории без Т-схемы! Но продолжай, вижу, тебе есть что еще сказать.

§2.5.6.3. Гупта. Критика дефляционизма

Иешуа: Третий аргумент я возьму из писаний гимнософиста Анила Гупты. [145] Дефляционизм обычно строится в два шага: (i) описание значения и функций истины; (ii) заключение от (i) к дефляционистским следствиям относительно истины. Иными словами, сторонники этого подхода предлагают анализ предиката «истинно», который состоит в том, что Т-схема говорит нечто важное о смысле термина «истинно», а потом утверждают: и это — все, что можно сказать об истине. С первым шагом их рассуждения мало кто стал бы спорить. Но как доказать второй шаг? Для этого дефляционистам пришлось бы показать, что выражения вроде «“р” — истинно, если и только если р» представляют собой случаи не просто материальной или необходимой эквиваленции, но эквиваленции по смыслу. Кажется, что этого сделать нельзя. Можно лишь критиковать другие теории, что дефляционисты и делают. Таким образом, дефляционизм либо тривиален, либо недостаточно обоснован. [146] Дефляционист в этой ситуации похож на недалекого стражника, который, поймав окровавленного человека с топором в руках рядом с трупом, заключает, что этот человек и есть убийца, и нет никакой нужды в дальнейшем расследовании.

Воланд: Думаю, с этим можно согласиться. Но ведь и противники дефляционистов должны показать случаи провала данного последними анализа. А они этого не делают: все приводимые ими примеры поддаются дефляционисткой переинтерпретации. То есть это тупик. Но в этой ситуации позиции дефляционистов выглядят более выгодными: они могут считать, что тождество выражений по смыслу есть тождество экстенсионалов этих выражений. Тождество экстенсионалов выражений «р» и «истинно, что р» никто не оспаривает. И если мы на основании экстерналистских [147] теорий значения приравняем смысл выражения и его экстенсионал, то получим устойчивый к возражению Гупты дефляционизм.

Иешуа: Как это не оспаривает! Именно что оспаривает. Если под экстенсионалом высказывания понимается какое-то положение дел, которое оно утверждает, то тождество «р» и «истинно, что р» отрицает Остин. Если под экстенсионалом высказывания понимается его истинностная оценка, то различие между этими выражениями показывает Даммит.

Воланд: Пусть так, хотя дефляционист может развить тут свою защиту и опровергнуть оксониенцев. Но дефляционист может настаивать и на другой мысли: бремя доказательств лежит на его противниках. Когда мы даем некоторое определение, отвечающее требованиям разумности и релевантности, это не мы должны показать, что никакая существенная черта определяемого предмета не осталась в темноте, а наши противники должны показать, что такие черты у предмета есть.

§2.5.6.4. Райт: инфляция дефляционизма

Иешуа: Но мы можем показать, что именно не удается объяснить дефляционисту! Заметим для начала, что в разговорах, ориентированных на истину, принято различать те обстоятельства, при которых принятие некоторых предложений обосновано, и те, в случае которых это не так. Если у меня будут иметься основания принимать, что p, то этот факт будет мне известен, и тем самым у меня будут основания утверждать, что у меня есть основания принимать p. Тогда (1) основания думать, что обоснованно утверждается, что p, имеются, если и только если есть основания думать, что p; (2) по схеме эквивалентности, которую дефляционисты признают: истинно, что p, если и только если p. Следовательно: (3) основания думать, что обоснованно утверждается, что p, имеются, если и только если есть основания думать, что истинно, что p. Теперь заметим, что обоснованная утверждаемость есть нормативное свойство, т.е. свойство, которое определяет, какие утверждения приемлемы, а какие нет. Приведенный довод показывает, что «основание думать, что p» и «основание думать, что истинно, что p» играют одну и ту же объяснительную, нормативную роль, они совпадают в позитивной нормативной силе. Этот факт относительно нашего способа утверждать высказывания не следует прямо из Т-схемы и является дополнительным фактом, который дефляционист должен признать, а объяснить в рамках своей схемы не может. [148]

Воланд: Как это не может!? Да это вообще не проблема для дефляциониста. Он скажет, что возможность перейти от (3) к (1) через Т-схему как раз и показывает, что истина прозрачна!

Иешуа: С прозрачностью истины я и не спорю! Я говорю, что у этой прозрачной истины есть еще черты, кроме тех, что фиксируются в Т-схеме!

Воланд: Но твой аргумент этого не показывает!

Иешуа: Как раз показывает! Давай еще раз!..

Пилат (прерывает его): Словом, тут опять требуются дальнейшие исследования. Я сужу так: этот человек отбился. Благодарю тебя, друг Луций. Теперь послушаем начальника стражи.

§2.6. Примитивизм

Dummett: Well you can say that, but the
question is what is the point of saying it?
Davidson: ah, eh... the point of saying is
that it makes some people happy.
(оба хохочут)

Марк: О могучий прокуратор! О достойная жена его! Вы знаете, что я старик честный, старик прямой. Я прямо говорю правду в глаза, даже если она неприятна... Позвольте мне сказать вам прямо, грубо, по-стариковски: пытаться дать определение истине просто глупо. Кто понимает, что такое истина, тот понимает; остальным — не объяснишь! [149] Таково мое солдатское мнение. Ваши философы все наводят тень на плетень, врут и завираются. А почему? А просто они прельстились речами Сократа об том, что нужно давать определения исследуемым понятиям! А между тем всякому доброму малому ясно, что философы обычно имеют дело с наиболее фундаментальными понятиями, и странно было бы ожидать, что мы сможем свести эти понятия к понятиям, которые будут еще более ясными и базовыми. Истина является как раз таким основополагающим понятием, так как именно от нее зависят понятия обоснованности, рациональности и значения. Анализируя то, как истина фундирует другие понятия, мы можем сказать много чего интересного об них, но не об ней самой. Истина — это простое понятие, не поддающееся никакому анализу или определению. [150]

Иешуа: Это, добрый человек, очень похоже на дефляционизм. А я...

Марк (прерывает его): Очень, да не очень! Дефляционисты сенатора правы в том, что истина — тривиальна. Но они ошибаются в том, что эта их Т-схема может задать смысл этого понятия. При этом с самой Т-схемой я согласен. И с тем, что все ее случаи истинны, я согласен. И с этим, с Хорвичем, тоже согласен, когда он говорит, что понимание истины связано с принятием любого конкретного случая Т-схемы как истинного без дальнейших свидетельств в его пользу. То есть я за критическую часть дефляционизма. Но я против его позитивной части.

Иешуа: Ты, кажется, сказал, что истина связана со значением. Не мог бы ты пояснить, как именно?

Пилат: Этот вопрос не имеет прямого отношения к делу, еще философию языка нам обсуждать тут не хватало! [151] Есть что обвиняемому сказать по существу?

Иешуа: Я думаю, что тут все просто. Если добрый человек Марк и его единомышленники обвиняют дефляционистов в том, что те не могут доказать свой позитивный тезис («об истине не нужно знать ничего, кроме Т-схемы и ее инстанциаций»), и на этом основании отвергают их подход, то я точно так же могу сказать, что они не доказывают свой последний шаг: они не приводят доказательства того, что истина как понятие проста и в ней больше ничего нет. Между тем я вижу большое достоинство подхода Марка: если нас интересует не истина, а другие философские понятия, то его подход является продуктивным — он позволяет вынести за скобки метафизику истины и сосредоточиться на других проектах. Но внутри исследований истины этот подход неприемлем.

§2.7. Алетический плюрализм

fruth
Alfred Tarski

Пилат: Прокла, возлюбленная жена моя, может быть, ты скажешь, что думаешь?

Прокла: Отчего нет, муж мой? Я скажу. Речь моя будет вослед тому, кто никогда не ошибается. Он, Афраний, уже говорил об идеях ребе Патнэма, когда представлял эпистемическую теорию истины как предела. Ты, Афраний, сказал, что этот учитель связывает истину с идеальными условиями исследования. Но сам же он потом опроверг этот подход, сказав, что никогда не имел в виду утопические требования Пирса, а имел в виду просто разумно хорошие условия исследования, которые у каждого высказывания свои. [152] Я согласна с Патнэмом. Иными словами, я думаю, что почти все вы правы и почти все вы ошибаетесь. Теоретики корреспонденции постулируют мистическую силу, связывающую слова и вещи. Они утверждают, что существует некое единое для всех случаев свойство «быть истинным», которое не зависит от контекста. Сторонники эпистемических теорий ошибаются, так как язык позволяет нам строить предложения, которые превосходят даже идеальную верифицируемость, — например, «не существует разумной жизни на далеких звездах», — и тем не менее мы понимаем, о чем в них говорится. Значит, истина не может быть сведена к верификации. Это одновременно является доводом и против дефляционизма: отрицая объяснительную роль истины, дефляционисты не могут объяснять значения высказываний через условия их истинности и должны объяснять их через верификацию. Однако корреспондентисты правы в том, что высказывания истинны или ложны в зависимости от положений дел в мире, а дефляционисты — в том, что утверждать, что пропозиция истинна, значит просто утверждать эту самую пропозицию. [153] Я предлагаю признать оба этих тезиса и объединить то, что в корреспондентизме и дефляционизме есть верного. Тогда наша теория будет состоять из следующих положений: (1) истина есть реальное свойство пропозиций, которое состоит в их правильности или ошибочности; (2) в каждом конкретном дискурсе (в этике, физике, религии, математике и т.п.) это свойство обладает собственной природой.

§2.7.1. Райт: истина как сверхутверждаемость

Прокла: Говорят, что чему-то подобному учит бритт Криспин Райт в Новом Эбораке, неподалеку от Хорвича, и даже называет он свое учение так же: минимализм, хоть это и совсем другой минимализм. Райт согласен, что понятие истины полностью схватывается Т-схемой, но он считает, что это еще не значит, что Т-схема есть все, что мы можем сказать об истине. Добрый человек из Назарета уже говорил о том, почему это так. [154] Именно этот довод и размышления о проблеме Мура [155] приводят Райта к понятию сверхутверждаемости, которое лучше всего выражает суть свойства «быть истинным». Высказывание является сверхутверждаемым, если и только если оно обосновано или может быть обосновано и если какое-либо из его оснований выдержит любое тщательное изучение происхождения этого основания. [156] Сверхутверждаемость, иными словами, тесно связана с познаваемостью. [157] Но это не значит, что она будет всюду выражаться одинаково: например, в моральном и эстетическом разговоре она будет чаще всего означать познавательные свойства оценочных суждений. В случае же высказываний о физическом мире сверхутверждемость будет зависеть от фактов, и в этом смысле от корреспонденции между словами и вещами.

Иешуа: Но ведь все это ведет к проблеме смешанных высказываний: [158] если в одном дискурсе истина обладает одной природой, в другом — другой, то мы не сможем рассуждать на пересечении этих дискурсов.

§2.7.2. Линч: функционалистская теория истины

Прокла: Именно так и полагает один из последователей Райта, Михаил Линч, преподающий свое учение в Коннектикутском государстве. Истина, учит он, относится к тем странным свойствам, которые называют функциональными. Свойство является функциональным, если оно позволяет своему обладателю выполнять некоторую функцию, при этом конкретный способ выполнения этой функции может отличаться в зависимости от различных обстоятельств. Возьмем, например, свойство «быть главой государства». Император Тиберий — глава государства, и император Серес — глава государства, и Артабан Парфийский — глава государства: у всех у них разные характеристики и полномочия, но все они выполняют одну функцию — вести свои народы к процветанию. «Истинность» относится к числу таких же свойств: как функциональное свойство оно едино для всех случаев, но в каждом конкретном контексте или дискурсе его выполняют, или манифестируют, разные свойства. Например, в юридическом контексте манифестирующим свойством для истины является когеренция некоего положения с корпусом нормативных документов, а в случае разговора об обыденных физических вещах, таких как чаши, сандалии и троны, манифестирующим свойством будет корреспонденция. Таким образом, хотя манифестирующих истину свойств много, функциональное свойство истины все же едино, и все манифестирующие его свойства отсылают к нему. А это значит, что проблема смешанных высказываний теряет свою хватку: все истинные высказывания в конечном счете истинны в одном смысле. [159]

Иешуа: Это похоже на попытку получить все и сразу и примирить всех врагов. Заманчиво, но плохо понятно, насколько надежны основания для принятия такой теории. Они были бы несомненно крепки, если бы удалось показать, что ни одна из традиционных теорий, которые Линч включает в качестве элементов в свою систему, не может справиться с объяснением истины во всех дискурсах в одиночку.

Прокла: Но именно такие основания Линч и приводит! [160] Так, например, он убедительно показывает, что твои любимые корреспондентисты не справляются с задачей объяснения того, как возможна истинность этических или математических высказываний; что здесь — границы применимости их теории. Помнится, ты и сам говорил о том, что нечто является истинным или ложным, если нечто другое, отличное от него, делает его истинным или ложным, разводя тем самым убеждения, об истинности которых мы можем говорить, и независимую от них и нашего ума реальность. Но очевидно, что соответствие убеждений и реальности в рамках корреспондентской теории мы должны объяснять не иначе, как через причинность. Ведь если бы корреспондентисты утверждали лишь то, что убеждение соответствует реальности, когда дела обстоят так, как о том свидетельствует убеждение, то с этим бы, по большому счету, готов был бы согласиться сторонник любой теории истины, и в таком случае корреспондентизм не заслуживал бы выделения в качестве некоторой отдельной позиции. Это значит, что репрезентационализм, предполагающий независимость реальности от ума и наличие между ними причинного отношения, составляет ядро корреспондентской теории. Но если корреспондентисты принимают репрезентационализм, то они сталкиваются с непреодолимыми проблемами объяснения того, как установить соответствие убеждений сущностям, которые не обладают каузальной эффективностью — например, абстрактным объектам, с которыми могли бы соотноситься положения математики; и как возможно соответствие убеждений сущностям, зависимым от нашего ума — например, этическим нормам. Все это как раз и говорит о том, что в данных областях корреспондентская теория оказывается бессильна. [161]

Иешуа: Твой Линч сильно спешит, ограничивая корреспондентизм одним лишь репрезентационализмом — кажется, я уже говорил о том, что сторонники теории соответствия не обязаны обременять себя этими дополнительными допущениями. [162] И в целом его доводы напоминают мне возражения игемона, которые я уже отводил в начале нашей беседы. [163] Я, впрочем, готов вновь повторить их целиком. Истина в том, что…

Пилат: Довольно! Истина в том, что мы уже ходим кругами. Дело идет к полудню. Мы увлеклись беседою, а между тем надо продолжать...

§3. Заключение

We are all fated to die
Peirce

На той неделе, наконец, с ФФ МГУ пришла запрошенная справка об истине; ознакомившись с ней, вновь сидел всю ночь над рукописью. Расследование зашло в тупик, и я не могу извлечь из доступных материалов более, чем следующего.

Из всех теорий истины в наиболее выгодной диалектической позиции традиционно находится корреспондентская. Именно с корреспондентской интуиции мы обычно стартуем, начиная разговор об истине, и именно ее держится большая часть как профессиональных философов, [164] так и разного рода филистеров. Она же получает заметное преимущество над многими конкурентами в ходе полемики.

Ряд традиционных теорий истины, теперь мне это совершенно ясно, проваливается под грузом критики. Когерентная теория не выдерживает аргумента двух систем. [165] Эпистемические теории опровергаются различными версиями проблемы Мура. [166] Теория тождества не справляется с проблемой лжи. [167] Примитивизм же представляет собой уход от проблемы природы истины, а не ее решение, пусть даже неудовлетворительное.

Вместе с тем дефляционизм и алетический плюрализм оказываются теориями, способными в своих наиболее прогрессивных формах (Хорвич и Линч соотв.) составить самую серьезную конкуренцию наиболее тонким версиям теории корреспонденции (Остин, Элстон и, возможно, теории факторов истинности). Все они в своих объяснениях природы истины выглядят не лишенными оснований, и успех той или иной зависит от ее возможности привести решающие критические доводы против остальных двух подходов. Однако несмотря на то, что определенные аргументы против своих противников теоретиками всех трех направлений были предложены, ни один из известных мне не может считаться решающим, и на данный момент борьба между корреспондентизмом, дефляционизмом и плюрализмом застыла в патовой позиции.

С другой стороны, можно задаться вопросом, что именно делает три перечисленных подхода (вне зависимости от их критического потенциала) одновременно столь привлекательными. Чем дальше, тем больше у меня создается впечатление, что ответ тут в том, насколько они — порой до неотличимости — похожи друг на друга.

В самом деле. Взять теорию корреспонденции, она же соответствия. Наименее привлекательными ее версиями являются варианты, в которых отношение соответствия задается «насыщенным» образом, например репрезентационалистские теории соответствия, объясняющие это отношение через причинность. Такие теории, как верно замечают плюралисты, весьма несложно раскритиковать. [168] Наиболее же перспективными и боеспособными версиями корреспондентизма представляются те, в рамках которых отношение соответствия задается минимальным, ненасыщенным образом, — а заданное таким образом отношение, как, опять же, справедливо замечают плюралисты, делает корреспондентскую истину малоотличимой от ее дефляционистской сестры. [169]

В то же время и сам алетический плюрализм, если посмотреть на него незамутненным взглядом, начинает сливаться с дефляционизмом. В самом деле, за обоими учениями стоят в существенной степени похожие интуиции: свойство истины многогранно, и для разных контекстов оно будет реализовываться по-разному. Отличие в том, что плюрализм пытается сохранить некую единую основу этого свойства и выделяет обозримое число дискурсивных областей, в рамках каждой из которых свойство может реализовываться своим образом. Дефляционизм же доводит эту последнюю плюралистическую интуицию до предела: где у плюралиста было обозримое число дискурсов, реализующих истинность по-своему, там у дефляциониста — бесконечное число частных случаев истинных пропозиций, в каждом из которых истина реализуется своим уникальным способом, так что разговор о каком-то общем свойстве истинности, которое бы объединяло все эти случаи, оказывается вообще лишенным смысла.

Возможно, в контексте таких принципиальных сходств выбор между этими вариантами — «слабыми» версиями теории корреспонденции, минимальным дефляционизмом и функционалистским плюрализмом — иллюзорен. Но сами их пропоненты, конечно, так не считают.

Что несомненно? В сухом остатке неиспарённым, видимо, остается только лишь то, что казалось верным уже и в начале: что «снег бел» — истинно, если и только если снег бел. Кажется, сегодня не существует хороших теорий, которые стали бы оспаривать схему эквиваленции Тарского (Т-схему): [170] это — та часть исследований истины, в которой все теоретики сходятся. Это значит, что, вероятно, каков бы ни был вывод грядущего ученого о природе истины, он может смело отталкиваться от выкристаллизировавшейся теперь формулы: «р истинно, если и только если р» или, короче,

T<p> ↔️ p.

И пусть кичливые буржуазные мыслители скажут, что все, чего мы добились в результате долгих исследований — это некий трюизм или, возможно, далеко не исчерпывающее определение истины; мы ответим им: мы нашли ее зерно — зерно истины, из которого произрастают любые ее побеги. Мало ли этого? На фоне мысли эксплуататорских классов, которую вечно, словно пыль, метают поветрия моды и где твердых выводов не бывает вовсе, наше зернышко представляется гранитной скалой. Хотя, конечно, не станем преувеличивать его значимости: в контексте общего урожая прогрессивной советской научной мысли оно является вполне скромным и закономерным результатом.

Что же до Воланда, то к его делу, видимо, нужно начинать искать подход с новой стороны…

Москва, Восьмая сестра, 2019 год

Примечания:

* Логинов Е.В., Юнусов А.Т., Мерцалов А.В., Саттар А.С., Басов А.С., Чугайнова Ю.И. Пролегомены к проблеме божественного существования // Финиковый Компот. 2017. № 12. С. 3–45.

Приложение.

Эволюция элиминации истины в дефляционизме

1. Дефляционизм начинается с теории избыточности, гласящей, что «истинно» можно элиминировать в любых контекстах: в частности, «истинно, что p» можно всегда заменить на «p». Смелый проект.

2. Уже у самого Рамсея он сталкивается с трудностями. Как элиминировать истину в контекстах вроде «Все, что сказал Маркс, истинно»? Просто убрать «истинно» тут нельзя. Поэтому Рамсей предлагает анализ вроде

(2) ∀х (М говорит, что хх)(i.).

Но формула (2), согласно стандартам логики предикатов, неправильно сформулирована. В качестве консеквента импликации в этом выражении встречается отдельно стоящая связанная переменная х — что грамматически невозможно. Дело тут в том, что поскольку эта формула сформулирована в логике предикатов первого порядка и поскольку мы, используя объектную квантификацию, квантифицируем по именным позициям, то х должно в ней везде занимать именную позицию. Но имя не может встречаться в предложении в одиночку: в подобной формуле оно должно быть дополнено каким-то предикатом — предположительно, как раз предикатом «истинно». (Рамсей чувствовал, что в его анализе есть некая проблема, но не считал ее серьезной. Еще Карнап использует для определения истины по существу этот рамсеевский логический ход (ii.).

3. Один из способов преодолеть эту проблему — изменить тип используемой при формализации нашего выражения квантификации: с объектной на подстановочную.

(3) ∀р (М говорит, что р → р).

В рамках подстановочной квантификации переменные не обязательно должны быть именами: мы можем квантифицировать по выражениям любого вида; в частности, в нашем случае — по предложениям. Поэтому выражение (3), в рамках которого мы квантифицируем по предложениям, является правильно сформулированным: р не нуждается в предикате, потому что он в р уже как в предложении содержится. Проблема, однако, в том, что подстановочная квантификация в классическом случае сама задается через истину: фактически выражение (3) читается как «всякий случай подстановки р в “M говорит, что р → р” — истинен». Если мы хотели избавиться от истины, то здесь нам это явно не удалось.

4. Просентенционалисты(iii.), следуя идеям Прайора(iv.), предлагают другое решение. С их точки зрения, объектная и заместительная не являются единственными видами квантификации. Прайор различает два значения выражения «переменная замещает». В первом переменная замещает выражение, на место которого она ставится: в случае объектной квантификации — имя, или единичный термин. Во втором смысле переменная замещает то, к чему осуществляло бы референцию выражение, на месте которого она стоит; в случае объектной квантификации — единичный объект. Однако, когда мы квантифицируем по предложениям (точнее, по позициям-предложений, sentence-places), переменная замещает нечто в первом смысле, но не во втором. Одновременно нельзя сказать, что в этом случае мы имеем дело с подстановочной квантификацией — истинностные условия квантифицируемых предложений не объясняются в терминах истины случаев их подстановки. Поэтому у нас есть третий вид квантификации: по позициям-предложений. И в выражении (3) мы используем именно его. Это, однако, влечет за собой определенные проблемы для нашего случая. Мы должны объяснить, как мы в рамках такой квантификации читаем

(3) ∀р (М говорит, что р → р).

Мы не можем прочитать эту формулу в подстановочном смысле: всякий случай подстановки предложения р дает истину — по указанным выше причинам. И мы не можем прочитать ее в объектном смысле: «Для всякого предложения, если Маркс говорит, что предложение, то предложение» — потому что это, как и раньше, не дает нам осмысленного высказывания. Просетенционалисты предлагают следующий способ прочтения (3): мы должны сказать, что р в данном случае — это неструктурированная просентенция типа «это-истинно». Эта просентенция должна работать на уровне объектного языка, потому что в противном случае термин «истинно» должны был бы быть отражен в формуле, сформулированной на метаязыке. Тогда (3) читается как:

(4) Для всякого предложения, если Маркс говорит, что оно-истинно, то оно-истинно.

Главная для нас проблема такой формализации связана с тезисом о неструктурированности просентенции. Этот тезис — единственный способ соблюсти отсутствие истины в метаязыке, поскольку если «это истинно» структурированно, то «истинно» должно читаться как предикат. Но есть существенные основания сомневаться в том, что «это-истинно» действительно является неструктурированной просентенцией — не в последнюю очередь таким основанием является то, что, исходя из утверждений самих просентенционалистов, это делает выражение «истинно» уникальным языковым феноменом, не имеющим аналогов в других выражениях. Просцентенционалисты пытаются держаться своей теории, преобразовывая ее под огнем критики тем или иным образом, но веры им все меньше. (Впрочем, не факт, что просентенционалисты действительно пытаются элиминировать истину. Они явно показывают, как она могла бы быть элиминирована в языке «английский*», являющемся фрагментом английского, в котором «это-истинно» рассматривается как неструктурированная просентенция. Но одновременно они, кажется, иногда признают, что в реальном английском «истинно» (по крайней мере отчасти) функционирует как предикат. Их элиминативистские амбиции неясны — прежде всего в силу в высшей степени характерного для этой теории напряжения между ее логическим и лингвистическим полюсами.)

5. Однако бόльшая часть дефляционистов в этом месте отказывается от амбиций по элиминации истины и признает, что предикат истины в подобного рода генерализациях неустраним из нашего языка. Поэтому (2) в действительности следует записать как

(5) ∀р (М говорит, что р → р — истинно).

Куайн подразумевает ровно это, говоря о семантическом восхождении(v.), однако в силу своей нелюбви к пропозициям и особенных отношений с теорией квантификации не дает эксплицитно этой формулы. Хорвич и симпатизирующие ему дефляционисты уже прямо используют именно формулы вроде (5)(vi.), утверждая, что здесь мы квантифицируем по пропозициям, которые можно рассматривать как объекты. Истина оказывается неустранимым предикатом; стратегии борьбы с ее «содержательностью» вынуждены сместиться с попыток ее элиминации на дефляцию других ее аспектов(vii.).

Примечания к приложению:

(i.) Рамсей. Факты и пропозиции. 2011. С.146–147.

(ii.) Carnap. Introduction to Semantics, 1946. P. 50ff.

(iii.) Grover, Camp, Belnap. A Prosentential Theory of Truth, 1975.

(iv.) Prior. Objects of Thought, 1971. Р. 35. См. также Grover. Propositional Quantifiers. 1972.

(v.) Куайн. Философия логики, 2008. С. 23–28.

(vi.) Horwich 19982, P. 4; 31–33.

(vii.) См. в целом: Dodd. An Identity Theory of Truth, 20082. P. 36–42.

Рассылка статей
Не пропускайте свежие обновления
Социальные сети
Вступайте в наши группы
YOUTUBE ×